Ich habe am Wochenende schon kurz erwähnt, dass jede Menge Gerüchte über eine “große Entdeckung” die Runde gemacht haben. Mittlerweile wurde bekannt gegeben, worum es sich handelt. Die wissenschaftliche Facharbeit dazu trägt den Titel “BICEP2 I: Detection of B-Mode Polarization at Degree Angular Scales” und das klingt ein klein wenig kompliziert. Und das ist es auch – aber es lohnt sich, sich mit diesem Thema ausführlich zu beschäftigen. Denn es geht um die Entstehung des Universums, seine Entwicklung und Wege, wie man all das untersuchen kann. Es geht um den Urknall, um Inflation, um Gravitationswellen und vor allem um “B-Moden”.

Das inflationäre Universum

Was also sind diese ominöse “B-Moden” und warum wäre ihre Entdeckung so wichtig? Um das zu verstehen müssen wir am Anfang anfangen und das buchstäblich. Hier geht es um Kosmologie, den Urknall und das, was unmittelbar danach im frühen Universum passiert ist. Das Urknallmodell ist heute allgemein als die beste Beschreibung der Entwicklung unseres Universums anerkannt. Entgegen weit verbreiteter Vorurteile handelt es sich dabei nicht um reine Spekulation, sondern um eine wissenschaftliche Theorie, die durch viele Beobachtungsdaten belegt ist. Ein integraler Teil des gegenwärtigen Urknallmodells ist die inflationäre Phase. Dabei handelt es sich um einen sehr kurzen Zeitraum, in dem die Expansion des Alls unmittelbar nach dem Urknall unvorstellbar viel schneller erfolgt ist als davor oder danach (Ich habe darüber früher schon mal ausführlicher gesprochen).

Urknall und Inflation (Bild: public domain)

Urknall und Inflation (Bild: public domain)

Die Inflation löst viele Probleme, die sich mit einem inflationslosen Universum nicht lösen lassen. Zum Beispiel das Horizontproblem: Dabei geht es um die Frage, warum das Universum so enorm homogen ist. Egal in welche Richtung man schaut, es sieht überall gleich aus (und man findet nicht zum Beispiel eine “Hälfte” des Universums die voll mit Galaxien ist und eine andere, die komplett leer ist). Das ist nur dann möglich, wenn sich auch die ursprüngliche Materie, die kurz nach dem Urknall das Universum erfüllt hat, entsprechend homogen verteilt war und das wiederum ist nur möglich, wenn diese Materie ausreichend Zeit hatte, sich genügend zu vermischen. Das junge Universum war zwar viel kleiner als heute, aber wenn man von einer normalen Expansionsrate ausgeht, dann war es trotzdem zu keinem Zeitpunkt klein genug als das das eine “Ende” des Universums wissen konnte, was im anderen “Ende” passiert. Die Inflation löst das Problem, in dem es davon ausgeht, dass das Universum eben nicht “normal” expandiert ist, sondern für einen kurzen Zeitraum quasi regelrecht “explodiert” ist. Zuerst war der Kosmos winzig und die Materie darin konnte sich problemlos vermischen und alles war homogen. Und dann blies die Inflation diesen winzigen Kosmos in unvorstellbar kurzer Zeit zu einem vergleichsweise riesigen Universum auf, das nun immer noch homogen war. Auch andere Probleme, wie zum Beispiel das “Flachheitsproblem” (die Frage, warum das Universum überall flach erscheint obwohl es eigentlich gekrümmt sein sollte) oder das Problem der magnetischen Monopole (die eigentlich vorhanden sein sollten, aber bis jetzt noch nicht gefunden werden konnten) werden durch die Inflation gelöst.

Das inflationäre Universum ist die bisher beste Beschreibung der Beobachtungsdaten die wir haben und die Theorie passt auch gut mit dem Rest der Kosmologie und Teilchenphysik (zum Beispiel dem kürzlich bestätigten Higgs-Mechanismus) zusammen. Es gibt bis jetzt viel indirekte Bestätigung für die Inflation aber noch keinen wirklich deutlichen direkten Beleg. Und genau hier kommen die primordialen B-Moden ins Spiel.

Hintergrundstrahlung

Wir haben nicht viele Möglichkeiten, das frühe Universum direkt zu untersuchen. In der Astronomie sieht man zwar um so weiter in die Vergangenheit, je weiter man in die Ferne blickt. Aber auch hier sind uns Grenzen gesetzt und der Vorhang, der die Geschehnisse im frühen Universum verdeckt heißt “kosmische Hintergrundstrahlung”. Ich habe in anderen Artikel – zum Beispiel hier – schon ausführlich erklärt, worum es sich handelt und möchte das daher hier nur nochmal sehr kurz ausführen.

1 / 2 / 3 / 4 / 5 / Auf einer Seite lesen

Kommentare (189)

  1. #1 BigBen
    17. März 2014

    “Lichtwellen können zum Beispiel in Ausbreitungsrichtung schwingen” -> Du wolltest sicher eine lineare Polarisation beschreiben, bei der die Welle transversal zur Ausbreitungsrichtung schwingt. 😉

  2. #2 Quantom
    17. März 2014

    Danke Florian,
    jetzt ist mir um einiges klarer geworden, was genau B-Moden sind.

    “…oder das Problem der magnetischen Monopole (die eigentlich vorhanden sein sollten, aber bis jetzt noch nicht gefunden werden konnten)..”
    Haben die Wissenschaftler nicht etwa magnetische Monopole in den letzten Monaten entdeckt oder erzeugt?!

  3. #3 Daniel Fischer
    Königswinter
    17. März 2014

    Was BICEP gefunden hat, werden wir bereits um 15:45 MEZ erfahren (die entsprechende Webseite ist z.Z. durch ein Password gesperrt) – das Paper inklusive.

  4. #4 Florian Freistetter
    17. März 2014

    @BigBen: “Du wolltest sicher eine lineare Polarisation beschreiben, bei der die Welle transversal zur Ausbreitungsrichtung schwingt.”

    Ja – und da stimmt das “in Ausbreitungsrichtung” doch? Die Welle schwingt senkrecht zur Ausbreitungsrichtung; also dorthin, wohin sich auch die Welle bewegt.

  5. #5 Florian Freistetter
    17. März 2014

    @Quantom: “Haben die Wissenschaftler nicht etwa magnetische Monopole in den letzten Monaten entdeckt oder erzeugt?!”

    Nein. Das waren nur gewisse Zustände, die wie Monopole aussehen. Magnetische Monopole sind gewissermassen Elementarteilchen – und die hat man noch nicht gefunden.

  6. #6 BigBen
    17. März 2014

    @Florian: Ja – und da stimmt das “in Ausbreitungsrichtung” doch? Die Welle schwingt senkrecht zur Ausbreitungsrichtung; also dorthin, wohin sich auch die Welle bewegt.

    Die Welle bewegt sich doch nicht senkrecht zur Ausbreitungsrichtung? #verwirrtdreinguck#

    BTW, vom Zusammenhang magnetischer Monopole uns Inflation habe ich noch nie gehört. Wäre das nicht mal einen eigenen Artikel Wert?

  7. #7 Florian Freistetter
    17. März 2014

    @BigBen: Ich zitiere jetzt mal Wikipedia: “Eine Transversalwelle – auch Quer-, Schub- oder Scherwelle genannt – ist eine physikalische Welle, bei der eine Schwingung senkrecht zu ihrer Ausbreitungsrichtung erfolgt.”

    Normalerweise erklär ich das immer mit nem Seil. Ein Ende ist festgemacht, das andere hat man in der Hand. Wenn man die Hand auf und ab bewegt kriegt man ne linear polarisierte Welle; wenn man die Hand im Kreis bewegt ne zirkular polarisierte Welle.

    “om Zusammenhang magnetischer Monopole uns Inflation habe ich noch nie gehört. Wäre das nicht mal einen eigenen Artikel Wert?”

    Mal sehen. Es gibt ja so viele Themen…

  8. #8 BigBen
    17. März 2014

    @Florian Ist schon klar. Aber Du sagtest “Die Welle schwingt senkrecht zur Ausbreitungsrichtung; also dorthin, wohin sich auch die Welle bewegt.” Die Schwingung ist senkrecht zur Ausbreitungsrichtung. Und die Bewegung der Welle wird durch den Wellenvektor beschrieben, der in Ausbreitungsrichtung zeigt und damit senkrecht zur Schwingung ist.

  9. #9 Florian Freistetter
    17. März 2014

    @BigBen: Ich wollte das mit der Polarisierung sehr anschaulich beschreiben; ohne irgendwelchen mathematischen Anspruch oder Wellenvektor. Und da erschien mir “Die Welle schwingt in die Richtung, in die sie sich auch ausbreitet – im Gegensatz zur zirkular polarisierten Welle, die spiralförmig um die Ausbreitungsrichtung herum schwingt” als vernünftige Beschreibung. Wenn ich ne Straße lang gehe und dabei immer auf und ab hüpfe, dann hüpfe ich “in Richtung der Straße” also “in Richtung meiner Ausbreitungsrichtung”.

  10. #10 Bullet
    17. März 2014

    Das ganze wird etwas verständlicher, wenn man im Kopf hat, daß Wellen prinzipiell ja auch longitudinal schwingen können. Und dann müßtest du auf der Straße (beim Bewegen aufs Ziel zu) vor und zurück rollern, während das Hüpfen ja auf und ab ist, also eben nicht “in Richtung der Straße”. diese Problematik ist aber schon in der Schule ohne anschauliches Vorturnen mit sich bewegenden Dingern am Lehrertisch schwer verständlich zu machen. Wie vieles in der Physik.

  11. #11 Alex
    17. März 2014

    Ich stimme da BigBen zu. Wollte das auch gleich als Kommentar posten und war froh, dass das schon jemand getan hat. Kurz: Eine Transversalwelle schwingt _senkrecht_ zu ihrer Ausbreitungsrichtung, wie Du ja selbst zitierst. Senkrecht zur Ausbreitungsrichtung ist aber gerade nicht _in Ausbreitungsrichtung_, wie Du im Artikel schreibst. Das wäre bei einer Longitudinalwelle der Fall.

    Eine andere Sache, die mich etwas irritiert hat, ist das erste Bild des Artikels. Du beschreibst die Inflationsphase des Universums ja als die Phase mit der schnellsten Ausdehnung. Das Bild aber legt nahe, dass die Inflation eine Phase langsamer Ausdehnung war, und erst danach, ungefähr da, wo im Bild “nucleosynthesis” steht, beginnt die eigentliche “Explosion”.

  12. […] Stories entnehmen, v.a. dem exzellenten Grundsatzartikel von 2001, oder späteren a hier, hier und hier. In der Inflations-Epoche des frisch entstandenen Alls wurden demnach Gravitationswellen […]

  13. #13 Franz
    17. März 2014

    Eine Grundsatzfrage: Bei Themen bezüglich der Hintergrundstrahlung wird immer nur von der Intensität gesprochen, aber niemals von der Frequenz. Ist die Frequenz eine Konstante ? Und wenn ja, dann müsste doch die originale Gammastrahlung auch eine konstante Frequenz haben ? Warum ?

  14. #14 Florian Freistetter
    17. März 2014

    @Bullet: “während das Hüpfen ja auf und ab ist, also eben nicht “in Richtung der Straße”. “

    Aber nur wenn ich im Stand hüpfe und nicht auch vorwärts. Dann hüpfe ich auch “in Richtung der Straße”. Aber Ok – dann korrigiere ich das mit der Ausbreitungsrichtung. Ich dachte, dass es so verständlicher wäre für Leute, die noch nie von Polarisation gehört haben…

  15. #15 Florian Freistetter
    17. März 2014

    @Franz: Die Wellenlänge der Strahlung nimmt zu, je weiter sich das All ausdehnt. Heute hat sie einen ganz konkreten Wert, bei dem sie beobachtet wird und bei diesem Wert misst man die Intensität.

  16. […] Was sind primordiale B-Moden und was haben sie mit Gravitationswellen, Inflation und dem Urknall zu … 17. März […]

  17. #17 Florian Freistetter
    17. März 2014

    Und das Gerücht wurde übrigens mittlerweile zur Gewissheit: https://www.preposterousuniverse.com/blog/2014/03/16/bicep2-updates/

  18. #18 Florian Freistetter
    17. März 2014

    @Alex: “und erst danach, ungefähr da, wo im Bild “nucleosynthesis” steht, beginnt die eigentliche “Explosion”.”

    Naja, die Inflation ist schwer zu visualisieren. Da wo du die “Explosion” ansetzt, beginnt die normale Expansion des Alls. Das die inflationäre Phase davor nicht so beeindruckend aussieht, liegt daran, dass du halt nicht visualisieren kannst, wie etwas aus einem quasi unendlich kleinen Punkt auf ein immer noch enorm kleines Objekt anwächst. Nach der Inflation war das Universum nach normalen Maßstäben immer noch winzig! Aber weil es eben vorher noch unvorstellbar viel winziger war, war es eben trotzdem ein ziemliches Ding! Aber das kann man halt alles schlecht visualisieren.

  19. #19 Franz
    17. März 2014

    @FF
    Heute hat sie einen ganz konkreten Wert, bei dem sie beobachtet wird und bei diesem Wert misst man die Intensität.
    Aber warum gerade diese eine Frequenz ? Da müsste doch die Originalstrahlung auch nur aus einer Frequenz bestanden haben. Wie geht das ? Primordialer Gammalaser ?

  20. #20 Florian Freistetter
    17. März 2014

    @Franz: “Aber warum gerade diese eine Frequenz ?”

    Naja, so viel Variation gabs damals nicht. Da gabs keine Dutzenden Elemente mit unterschiedlich konfigurierten Elektronenhüllen. Da gabs Protonen und von denen hat sich jedes ein Elektron geschnappt und das hat bei der Rekombination Strahlung ausgeschickt. Ok, ein paar Heliumkerne waren auch mit dabei und das wird schon ein kleines bisschen Variation erzeugt haben. Aber das wars dann auch schon wieder.

  21. #21 Alderamin
    17. März 2014

    @Franz, Florian

    Die Hintergrundstrahlung hat doch gar nicht nur eine Frequenz, sondern es handelt sich um ein Schwarzkörper-Kontinuum, und zwar eines, das einer Temperatur von ca. 3000 K entspricht, das um den Faktor 1000 rotverschoben ist (mit dem linearen Wienschen Verschiebungsgesetz kommt man so auf ca. 3 K der beobachteten rotverschobenen Strahlung). Also handelt es sich um ein Gemisch zahlreicher Frequenzen mit einer charakteristischen Planckkurven-Verteilung.

    Es ist genau diese Temperatur, weil bei dieser Temperatur das Plasma neutral wurde: die freien Elektronen wurden von Atomkernen (zu 75% einfache Protonen: Wasserstoffkerne) eingefangen und Licht beliebiger Frequenz wurde nicht mehr absorbiert und gestreut (wie das bei den freien Elektronen im Plasma der Fall war) sondern nur noch solches, das die Frequenz der Spektrallinien des Wasserstoffs (bzw. Heliums) hatte. Somit wurde das neutrale Gas praktisch vollkommen transparent.

    Durch das (jüngere) transparente Gas blicken wir also auf ein (älteres) Plasma, das an seiner Außenseite eine Temperatur von 3000 K hat.

    Ähnliches beobachtet(e) man übrigens bei Kernwaffenexplosionen (“Breakaway”).

  22. #22 Franz
    17. März 2014

    @Alderamin
    Danke für die Erklärung, jetzt hab ich es verstanden. Beim Begriff ‘Intensität’ habe ich irrtümlich an eine einzige Frequenz (und dessen Amplitude) gedacht und nicht ein ganzes Spektrum.

    Nicht umsonst heißt es ja auch ‘Hintergrundrauschen’ und nicht ‘Hintergrundpfeifen’ 🙂

  23. #23 Lili
    17. März 2014

    Noch eine Stunde bis zur Pressekonferenz, und schon in der Server vom Harvard CfA in die Knie gezwungen worden – na das fängt ja gut an. Jetzt hab ich nicht mehr viel Hoffnung dass ich das heute live sehe 🙁

  24. #24 stone1
    17. März 2014

    Ah ja, die Entdeckung ist also schon durchgesickert (#17).
    Bin dennoch gespannt auf die Pressekonferenz, hoffentlich läuft das dann auch, bisher führt der entsprechende harvard.edu-Link noch nirgendwohin.

  25. #25 Lili
    17. März 2014

    ja die bei speceref, die das ganze losgetreten haben, haben anscheinend auch schon Gewissheit: https://spaceref.com/astronomy/first-direct-evidence-of-cosmic-inflation.html
    Aber dass denen die Leute vom CfA auch schon Interviews gegeben haben, finde ich sehr suspekt, das kann doch irgendwie nicht sein. Wenn das eigene Institut eine Pressekonferenz ankündigt, dann kann (darf!) man vorher nicht einfach den Medien Auskunft geben, jedenfalls nicht ohne mächtig Ärger zu bekommen. Hmm

  26. #26 Alderamin
    17. März 2014

    r=0,2

    Und was heisst das jetzt für die Inflation?

  27. #27 stone1
    17. März 2014

    @Alderamin:
    Vielleicht findet man da drin eine Antwort, ich bin selber aber grad viel zu zappelig um mich da reinzulesen.

  28. #28 Nico
    17. März 2014

    @Florian: Ist diese Entdeckung auch eine weitere Bestätigung für den Urknall aus einer Quantenfluktuation?

  29. #29 Florian Freistetter
    17. März 2014

    @Nico: Naja, das es den Urknall gegeben hat war auch davor schon bestätigt; genauso wie die Existenz er Quantenfluktuationen (siehe “Kasimir-Effekt”). Aber das ist die erste direkte Bestätigung der Inflation und das ist wirklich cool!

  30. #30 Alderamin
    17. März 2014

    Schöner Artikel (via Skyweek), der auch klar macht, warum Gravitationswellen eine Polarisation verursachen: sie strecken den Raum in einer Richtung und komprimieren ihn senkrecht dazu. Durchlaufendes Licht wird somit polarisiert (in einer Sky & Telescope von neulich war auch erklärt, warum das eine zirkulare Polarisation ergeben sollte, ich hab’ das aber nicht mehr so genau im Kopf).

  31. #31 Theres
    17. März 2014

    Matt Strassler, Daniel Fischer und Universe Today “leaken” ebenfalls, und damit scheint es sicher zu sein. Na, das ist ja was … jetzt muss ichs nur noch verstehen!

  32. #32 Alderamin
    17. März 2014
  33. #33 Florian Freistetter
    17. März 2014

    Der Artikel ist jetzt übrigens aktualisiert und mit den neuen Infos ausgestattet.

  34. #34 Nico
    17. März 2014

    “Aber das ist die erste direkte Bestätigung der Inflation und das ist wirklich cool!”

    Und somit der erste Beweis eines Quantenphänomens zum Zeitpunkt des Urknalls?

  35. #35 stone1
    17. März 2014

    Tja, das war dann wohl vorerst nix mit der Live-Pressekonferenz. Ich bekomme zumindest keine Verbindung, da scheint’s tatsächlich gröbere Serverprobleme beim Harvard CfA zu geben.

  36. #36 Alderamin
    17. März 2014

    @Nico

    Die Inhomogenitäten in der Hintergrundstrahlung und die großräumige Struktur des Universums mit Voids und Bubbles wurden schon immer mit durch die Inflation hochskalierte Quantenfluktuationen in Zusammenhang gebracht, aber dies ist wohl jetzt die erste echte unabhängige Bestätigung dafür.

  37. #37 Nico
    17. März 2014

    Diese Alternative funktioniert: https://www.ustream.tv/channel/aagie

  38. #38 MisterX
    17. März 2014

    Und jawooohl genau richtig gelegen ! Inflation ist nun richtige empirische Wissenschaft 😉 MUHAHAHAHA

  39. #39 Nico
    17. März 2014

    @Alderamin: Ich dachte immer, diese Inhomogenitäten etc. wären durch die Dunkle Materie entstanden und würden halt Dichteschwankungen darstellen?

  40. #40 MisterX
    17. März 2014

    Das heisst Gravitationswellen indirekt gemessen und Inflation !!! HAMMEER

  41. #41 MisterX
    17. März 2014

    Higgs war gestern 😀

  42. #42 Alderamin
    17. März 2014

    @Nico

    Nö, die dunkle Materie hätte ja auch erst mal homogen verteilt sein müssen, wenn es keine Dichteinhomogenitäten gegeben hätte. Sie hat den Kollaps der Materie lediglich beschleunigt und die Strukturen gefestigt.

    Siehe z.B. hier, Kapitel V. Im folgenden steht auch was zum Einfluss der dunklen Materie.

  43. #43 stone1
    17. März 2014

    @Nico:
    Danke, hab nur die ersten paar Minuten versäumt.
    Ein Gedanke der mir dabei durch den Kopf ging:
    Eat this, creationists! 😉

  44. #44 Alderamin
    17. März 2014

    @stone1

    Zumindest die katholische Kirche hatte nie Probleme mit der Urknalltheorie. Der Entdecker des expandierenden Universums, Georges Lamaître, war sogar Priester. Die Theorie passt doch schön zur Bibelstelle: “Und Gott sprach: Es werde Licht! Und es ward Licht.”

    Interessant wäre allerdings, wenn man jetzt anhand der gefundenen Messwerte zeigen könnte, dass die Inflation nicht überall komplett enden kann, weil es dann eine unendliche Zahl von Universen geben müsste. Da hätte die Kirche dann mehr zu schlucken, weil wir damit statistisch unvermeidlich und gänzlich unbedeutend im Vergleich zum Ganzen wären – sicher nicht die “Krone der Schöpfung”.

  45. #45 Florian Freistetter
    17. März 2014

    @Alderamin: Mit dem Urknall kommt die Kirche vielleicht klar. Aber auch mit dem Multiversum? Denn das hängt in der Inflationsgeschichte ziemlich stark drin. Wenns Inflation gab, dann ist die Wahrscheinlichkeit groß, dass es auch das Multiversum gibt. Mal sehen, vielleicht schreib ich da morgen was dazu.

  46. #46 stone1
    17. März 2014

    So, das war’s. Mann jetzt ärgere ich mich, dass ich erst gegen Ende dran gedacht hab, den SBX-ProStudio-Crystalizer zu aktivieren. Das hat die schlechte Audioqualität gut kompensiert.

    @Alderamin:
    Naja, wie bei der Frage zur Multiversumtheorie ja gesagt wurde, das ist erst der Anfang. Vielleicht kann man sich mit dieser Eingrenzung aber in Zukunft tatsächlich Experimente zum Nachweis von anderen Universen ausdenken, mein Gedanke war auch nicht so ernst gemeint.

    Eine super Leistung haben die Wissenschaftler vom Smithsonian Center jedenfalls abgeliefert, und man wird sich wohl bald auf ausführlichere Meldungen, Artikel und Bücher zum Thema freuen dürfen.

    Aber jetzt muss ich mich erst mal wieder um anderer Leute IT-Probleme kümmern.

  47. #47 Zhar The Mad
    17. März 2014

    Ist es eigentlich für die Theorien zwingend nötig, dass das Universum “unendlich klein” begonnen hat, und nicht etwa schon “enorm klein”? Womit eine inflation an sich nicht mehr nötig wäre, da ja schon von anfang an “klein” genug?
    und jetzt bitte keine fragen wie es denn schon klein hätte beginnen können, denn die frage des “was war davor” ist immer ungeklärt mir geht es wirklich nur darum, ob es wirklich aus den Theorien zwingend hervorgeht, dass es unendlich klein begonnen haben muss, oder obs einfach nur “schön” zum rest passen würde. Wens nur schönes beiwerk, ohne speziellen vorhersagecharakter ist, warum dann behaupten?

  48. #48 Florian Freistetter
    17. März 2014

    @Zhar: Die Inflation braucht es auf jeden Fall; egal ob es “unendlich” oder “enorm” klein ist. “Unendlich” klein wird es wohl nicht gewesen sein, denn Unendlichkeiten existieren in der Natur normalerweise nicht. Eher “so verdammt klein, dass man es sich nicht vorstellen kann und es für uns keinen Unterschied macht, ob es jetzt unendlich klein oder nur verdammt klein ist”.

    Wenn du mehr zu dem Thema wissen willst, dann empfehle ich dir dieses Buch: https://scienceblogs.de/astrodicticum-simplex/2010/05/31/der-stoff-aus-dem-der-kosmos-ist-die-komplette-rezension/

  49. #49 Jürgen Schönstein
    17. März 2014

    Na, das freut mich für meinen Kollegen (im Sinn von “ebenfalls am MIT beschäftigt Seienden”) Alan Guth. Ich hatte mehrfach Gelegenheit gehabt, mit ihm in einer zwanglosen Runde zu plaudern, und kenne keinen angenehmeren Menschen, dem ich einen Nobelpreis wünschen würde.

  50. #50 Alderamin
    17. März 2014

    @Zhar The Mad

    Möglicherweise war das Weltall sogar von Anfang an unendlich groß. Die Inflation bzw. die Urknalltheorie sagt nur etwas über das Wachstum eines Volumenelements des Raums aus, nichts darüber, wie groß das Volumen insgesamt war.

    Ein unendliches Anfangsvolumen ist sogar das Standardmodell. Wie das funktionieren soll, wird in diesem Artikel erläutert.

    Insofern kann man nur sagen, dass das beobachtbare Universum sehr klein begann, aber nicht das Universum insgesamt.

  51. #51 Quantom
    17. März 2014

    Florian Freistetter
    17. März 2014

    @Florian:
    “… Wenns Inflation gab, dann ist die Wahrscheinlichkeit groß, dass es auch das Multiversum gibt. Mal sehen, vielleicht schreib ich da morgen was dazu. …”

    Oh ja, bitte (es muss ja nicht gleich morgen sein), darüber würde ich mich echt informieren, wieso es ausgerechnet an der Inflation davon abhängt, dass es auch das Multiversum gibt. 🙂

  52. #52 AP
    17. März 2014

    “Die verborgene Wirklichkeit” von Brian Greene (kürzlich wieder gelesen) geht detailliert auf das Thema Inflation / Multiversum ein. Ich finde es sehr aufregend, dass es nun tatsächlich starke Indizien für diese Hypothesen gibt!

  53. #53 Florian Freistetter
    17. März 2014
  54. #54 MisterX
    17. März 2014

    Oh man und nächstes Jahr geht der LHC auf 17 Tev ! Was nochmal viel höher ist als er ansonsten war. Dann gehts bestimmt gleich weiter mit neuen Erkenntnissen 😀 JAWOHL !

  55. #55 afx
    17. März 2014

    Ohmann, bin ich froh, dass es diesen Blog gibt.
    Was da so in anderen Kommentarbereichen, besonders SPON, verbreitet wird, lässt einem die Haare zu Berge stehen.

    Und JEDES mal kommen diese Anti-Wissenschafts-Trolle aus ihren Löchern gekrochen nach dem Motto “Was-bringt-das-der-Menschheit?-NICHTS!” und “Mit-dem-verpulverten-Geld-hätte-man-so-viele-Menschen-vorm-Hungertod-retten-können”.

    DANKE FLORIAN! 😀

  56. #56 Alderamin
    17. März 2014

    @afx

    Ich geh’ mich auch immer im SpOn-Forum gruseln und kann dann mein Entsetzen nicht fassen. Und rede mir dann immer ein, das ist nicht repräsentativ, das ist nicht repräsentativ, das KANN NICHT repräsentativ sein. Nicht niemals.

  57. #57 aphex
    Berlin
    17. März 2014

    @afx

    Hab ich auch grad so bei mir gedacht..
    Also auch von mir ein grosses Dankeschön, Florian!

  58. #58 Florian Freistetter
    17. März 2014

    @afx, aphex: Gern geschehen! Mach ich gerne. Ist ja auch der Beruf mit dem ich mein Geld verdiene (*hust* flattr-Button *hust*) 😉

  59. #59 Alderamin
    17. März 2014

    Im Heute-Journal surft Einstein auf den heute entdeckten Gravitationswellen. Von der Inflationstheorie kein Wort. 🙁

  60. #60 PDP10
    17. März 2014

    Wow! Cool! Cool! Cool!

    @Florian:
    Ich kann mich nur @afx und @Alderamin anschliessen:

    Ich bin froh, das du das hier in aller Ausführlichkeit – und verständlich! – erklärt hast.
    Well done!

    Die Meldung ging tatsächlich heute durch alle grossen Online-Medien von SPON über Zeit-Online bis FAZ.net.
    War aber nirgendwo wirklich vernünftig erklärt.

    Und ich kann mich den Vorpostern auch nur anschliessen was die Qualität der Postings in den Foren dort angeht …

    Wenn man jemals einen Grund gebraucht hat um zu glauben, “das es eine gute Idee wäre, die Sache mit der Evolution nochmal von vorne anzufangen” (Douglas Adams) dann braucht man sich nur die 26+ Seiten Kommentare bei SPON durchzulesen …

    Du mei-ne-gü-te! ….

    Und die “Wozu soll das gut sein” Fraktion ist noch harmlos …

  61. #61 Hans
    17. März 2014

    #56 Alderamin

    @afx

    Ich geh’ mich auch immer im SpOn-Forum gruseln und kann dann mein Entsetzen nicht fassen.

    Du scheinst also auch eine gewisse… mmhhh… masoschistische Ader zu haben, oder? – Oder warum tust Du Dir so ein Forum an, wenn Du anschliessend vor entsetzen nicht fassen kannst, was Du dort findest?

    Und rede mir dann immer ein, das ist nicht repräsentativ, das ist nicht repräsentativ, das KANN NICHT repräsentativ sein. Nicht niemals.

    Nun gut, dann wäre zu prüfen, ob das nur eine Hoffnung Deinerseits ist, oder ob die grosse Mehrheit tatsächlich eher das glaubt, was solche Leute schreiben, die man besser als Meinungsmacher bezeichnen sollte.

  62. #62 Theres
    17. März 2014

    Ich war auch kurz im SpOn- forum und waaah! Schließe mich den Vorpostern umfänglich an, vor allem mit dem Dank an Florian – nur nicht Hans – ich bin nämlich keineswegs maso, nein, nur neugierig …
    @Alderamin
    Tja, das Zweite halt – so ganz altersgerecht war das aber nicht mit dem Gravitationssurfer … wait, Silversurfer 🙂

  63. #63 Nico
    17. März 2014

    @Alderamin und Florian

    Aber die aufgeblasenen Quantenfluktuationen kann es doch nur zur Zeit der Planck-Ära gebeben haben, oder? So habe ich es mir zumindest bisher vorgestellt.

  64. #64 stone1
    18. März 2014

    Die SpOn Kommentare sind wohl genauso wenig repräsentativ wie die bei Scienceblogs, und der Großteil der Menschheit wird die Entdeckung der inflationären Gravitationswellen mittelfristig wohl mit einem ähnlich achselzuckenden “Na schön dass man das jetzt weiß, dann können wir uns ja wieder dem Alltag widmen” hinnehmen wie beim Higgsteilchen, aber hier ist man wenigstens gut informiert.

    Und das ist doch alles was zumindest für mich zählt. Ein weiteres Puzzlestück im großen Rätsel namens Universum – oder eben Multiversum – wurde gefunden. Jetzt wird man sich daran machen, es an die richtige Stelle zu setzen. 🙂

    Ich bitte die etwas schmalzige Ausdrucksweise zu entschuldigen, sowas passiert mir manchmal.

  65. #65 stone1
    18. März 2014

    Nachtrag:
    Es war wie die Suche nach der Nadel im Heuhaufen. An deren Stelle haben wir aber eine Brechstange gefunden.

    Das war für mich das Zitat der Pressekonferenz. Aber jetzt tanzt mir wieder so ein Brechstange schwingender Gordon Freeman durch den Kopf. 😉

  66. #66 JoJo
    18. März 2014

    Ich suchte eben im gesamten deutschsprachigen Internet nach einem Artikel der mir erklärt, was es damit genau auf sich hat.

    Nur hier wurde ich endlich fündig.

    Vielen Dank dafür 🙂

    Ich finde das sehr interessant.

    Meine Frage, die ich mir schon seit 20 Jahren stelle wird zwar auch nicht beantwortet, muss sie nicht, wohl auch nicht: und wird sie wohl auch nie.
    Was soll das alles eigentlich, was hat es zu bedeuten und warum ist überhaupt was und nicht Nichts?
    Und wieso gab es den Urknall überhaupt, wieso blieb es nicht für alle Ewigkeit (auch wenn man von Zeit nicht sprechen kann) eine Singularität. Das, was es eben “vor” dem Urknall war.

    Auch wenn die Frage ein bißchen sinnlos ist bleibt sie für mich unglaublich spannend.

  67. #67 Bullet
    18. März 2014

    Gordon doesn’t need to hear this. He’s a highly trained professional.

  68. #68 PDP10
    18. März 2014

    “Aber jetzt tanzt mir wieder so ein Brechstange schwingender Gordon Freeman durch den Kopf”

    Sowas ähnliches ging mir bei dem Satz auch durch den Kopf – obwohl ich Gordon Freeman nur vom Hörensagen kenne (hab HL nie gespielt) …

    Los! Rück raus damit Universum!!!

    *KATCHUNK*

  69. #69 ulfi
    18. März 2014

    Wie passt das denn zu den anderen Messungen in der Grafik? Die Mssung ist ja jetzt nun ziemlich weit ausserhalb der Konfidenzintervalle der anderen Verfahren. Macht das noch Sinn? Hat man den Messfehler so weit unterschätzt?

  70. #70 McPomm
    18. März 2014

    Kann man das vielleicht so zusammenfassen?
    Alan Guth hat in seiner Inflationstheorie u.a. vorhergesagt, dass dabei Gravitationswellen entstehen und diese das Licht (das erste sich frei ausbreitende Licht, das ist die kosmische Hintergrundstrahlung) B-modal polarisiert. Das BICEP-2-Team hat sich gesagt, dass sich diese Polarisierung im Muster der minimalen Helligkeitsschwankungen der Hintergrundstrahlung in ganz bestimmten wirbelförmigen Mustern niederschlagen müsste. Und genau diese Muster wurden jetzt gefunden. Oder?

  71. #71 Trottelreiner
    18. März 2014

    @alderamin, 56:
    Auf die Gefahr einen Godwins hin…

    “Mein Sohn, wir sind Pilger in einem gottlosen Land.”

  72. #72 Florian Freistetter
    18. März 2014

    @ulfi: “Wie passt das denn zu den anderen Messungen in der Grafik? “

    Dazu schreibe ich heute noch etwas.

  73. #73 J. Mischke
    18. März 2014

    Eine ganz dumme Frage, die sich immer aufdrängt. Wenn ALLE Materie das Universums bereits vorhanden war ( eben auf einem kleinen Punkt konzentriert ) muss doch eine unwahrscheinliche Gravitation geherrscht haben.
    Das ist ja nun wieder einem schwarzen Loch nicht unähnlich.
    Aber ein Ereignishorizont des schwarzen Loches setzt wiederrum Raum vorraus, der damals ja noch gar nicht vorhanden war.
    Frage: Wie konnte das Unversum gegen die Schwerkraft ankämpfen?

  74. #74 Florian Freistetter
    18. März 2014

    @J. Mischke: “Wie konnte das Unversum gegen die Schwerkraft ankämpfen?”

    Das kann man leider nicht mal so eben schnell erklären. Da gehts um Quantengravitation, Vereinheitlichung der Kräfte, Inflaton-Felder, etc. Simpel gesagt: Das frühe Universum verhielt sich physikalisch VÖLLIG anders als das heutige. Ich empfehle dazu immer dieses Buch: https://scienceblogs.de/astrodicticum-simplex/2010/05/31/der-stoff-aus-dem-der-kosmos-ist-die-komplette-rezension/

  75. #75 mmaker
    18. März 2014

    Können Sie bitte etwas über die Graphik im Blog sagen? Was sind die blauen und was die gelben Felder? Und wenn man so draufguckt – ich kann es leider nicht genauer formulieren – wo befindet man sich auf der Erde und wohin guckt man dann? Sie haben oben irgendwo geschrieben, dass die Hintergrundstrahlung konstant/gleichförmig/oder so ist, warum dann die verschiedenen Felder in der Graphik? Vielen Dank für Ihren Blog und auch für Ihre Antworten auf die Kommentare der Leser.

  76. #76 Markus Pössel
    Heidelberg
    18. März 2014

    Nur ein kleines Detail, aber: das Südpolteleskop, das du nennst und zeigst, ist soweit ich das verstanden habe nicht das Teleskop, was die Messungen gemacht hat. BICEP2 ist ein eigenes Teleskop, oben auf dem Gebäude, von dem in deinem Bild gerade noch ein Stück ins Bild ragt.

    Auf diesem Bild hier:

    https://www.scilogs.de/relativ-einfach/bicep2-und-die-ersten-sekundenbruchteile-nach-dem-urknall/#bicep

    ist BICEP die Schüssel rechts auf dem Dach; das South Pole Telescope ist das Radioteil links.

  77. #77 Florian Freistetter
    18. März 2014

    @mmaker: “Können Sie bitte etwas über die Graphik im Blog sagen? Was sind die blauen und was die gelben Felder?”

    Welche Grafik meinen sie? Die mit der kosmischen Hintergrundstrahlung? Die zeigt den kompletten Himmel – die Erde befindet sich dort also nirgends. Es ist kein direktes “Foto” des Himmels, sondern eine Karte und die zeigt das, was sie sehen könnten, wenn ihre Augen Mikrowellen sehen und enorm empfindlich wären. Die Hintergrundstrahlung IST gleichförmig, weist aber trotzdem winzige Variationen auf. Und genau die zeigt die Grafik.

  78. #78 Florian Freistetter
    18. März 2014

    @Markus: Ich dachte “Südpolteleskop” heißt der ganze Komplex dort? So wie die “Südsternwarte” der ESO, die ja auch aus vielen Teleskopen/Instrumenten besteht… Aber danke für den Hinweis!

  79. #79 mmaker
    18. März 2014

    @Florian : Genau die Grafik meinte ich. Dass die Erde da sich nirgends befindet ist klar, aber schaut man von Greenwich aus in den Himmel oder vom Südpol aus? Was mich verwirrt ist, dass die Grafik eine Form hat wie Weltkarten, also müsste ein gewisser Bezug zur Erde da sein.

    Eigentlich hätte ich auch zu einer Grafik, die im Artikel von Science gezeigt wurde, wo man diese Wellen, um die es hier so geht, _am Rand_ als schwarze Flecken sehen soll. Diese Grafik hat auch so eine Weltkartenform, und man sieht da drauf auch wirklich ein paar schwarze Bereiche am Rand.
    Nach meinem Verständnis dieser Wellen sollten die doch eher überall und nicht nur am Rand auftauchen. (Ich such mal den Link zu dieser Grafik heraus.)

  80. #80 mmaker
    18. März 2014

    Ich hab leider den Artikel nicht gefunden, aber auf
    https://bicepkeck.org/
    gibt es unter diesem Link
    https://bicepkeck.org/B2_2014_i_figs/tqu_maps.png
    die Grafik im linken obersten Feld.
    Sehen Sie die schwarzen Ränder am Rand der wabernden roten und orangenen Felder, _die_ sollen die jetzt gefundenen Wellen sein.

  81. #81 McPomm
    18. März 2014

    @mmaker:
    Die Karte von der Kosmischen Hintergrundstrahlung ist die Weltkarte quasi von innen nach außen betrachtet, also eine astronomische Weltkarte. Man könnte z.B. Sternbilder einzeichnen. Wenn ich das richtig damals verstanden habe, wurde der Beitrag der Milchstraße herausgerechnet. Aber nicht der Beitrag von entfernten Galaxien/Galaxie-Clustern. Hingegen wurde bei BIZEP2 nur ein Teil des Himmels beobachtet. Und es wurde zudem noch der Beitrag von Galaxien, Galaxie-Clustern und vor allem kosmischem Staub herausgerechnet, eben die nicht-primordialen Anteile an der Polarisation (gerade, was den Staub angeht, gibt es Diskussionen).

  82. #82 Nico
    18. März 2014

    @mmaker:

    Diese Karte beantwortet sehr schön deine Frage: https://sci.esa.int/science-e-media/img/db/Planck_FIRST_LIGHT_SURVEY_hi.jpg

    Zum einen erkennst du den Himmel, wie man ihn mit bloßen Augen sieht; in der Mitte ist das Milchstraßenzentrum Richtung Schütze/Skorpion zu sehen. Darüber wurde ein schmaler Streifen gelegt, den du erkennen würdest, wenn deine Augen empfindlich für langwellige Radiowellen wären. Mit entsprechender Auflösung detektieren Radioteleskope eben ein fleckiges Muster, das einfach in einer beliebigen Farbe dargestellt wird. Das ist die Hintergrundstrahlung.

  83. #83 Lili
    18. März 2014

    ich grüble immer noch über den Titel der Fachpublikation: “…at degree angular scales” – fehlt da nicht etwas? Ich würde verstehen wenn da stehen würde “high degree” oder “low degree” oder “five degree”, aber was ist “…at degree angular scales”? Kommt wohl davon wenn es noch gar kein paar-review gegeben hat.

  84. #84 Lili
    18. März 2014

    blöder Tippfehler: peer-review natürlich

  85. #85 Klaus
    18. März 2014

    “und das klingt ein klein wenig kompliziert. ”
    .
    “Ein klein wenig” ?
    Ich versteh’ nur Bahnhof.

  86. […] habe gestern schon ausführlich über die wissenschafliche Hintergründe geschrieben und erklärt, was die Inflation ist, warum man diese These entwickelt hat, wie man sie […]

  87. #87 Florian Freistetter
    18. März 2014

    @Klaus: “Ich versteh’ nur Bahnhof.”

    Schade. Ich hab mir große Mühe gegeben, halbwegs verständlich zu erklären, worum es geht. Aber leider fehlt mir hier im Blog die Zeit und der Platz, wirklich bei Null anzufangen und alles komplett ausführlich zu erklären. Dazu bräuchte es ein kleines Buch…

  88. #88 Florian Freistetter
    18. März 2014

    @Lili: “Ich würde verstehen wenn da stehen würde “high degree” oder “low degree” oder “five degree”, aber was ist “…at degree angular scales”?”

    Hmm. Ich habe das so interpretiert, dass man halt auf der Größenordnung von nem Grad hingeschaut hat. So wie halt “Meter” im Vergleich zu “Kilometer” oder “Zentimeter” – denn die primordialen B-Moden sind ja nur auf diesen großen Skalen zu sehen. Aber vielleicht ist es wirklich auch nur ein Tippfehler (dann würde der aber nicht überall sonst auftauchen).

  89. #89 J. Mischke
    18. März 2014

    Danke, hochinteressante und verständliche Erklärungen. Das Buch von Green werde ich mir mal holen.
    Nochmal DANKE!

  90. #90 Florian Freistetter
    18. März 2014

    @mmaker: “Sehen Sie die schwarzen Ränder am Rand der wabernden roten und orangenen Felder, _die_ sollen die jetzt gefundenen Wellen sein.”

    Das ist die Messung der Polarisation der Hintergrundstrahlung von der ich im Artikel gesprochen habe. Die wurde durch die Gravitationswellen verursacht.

  91. #91 mmaker
    18. März 2014

    Florian, McPom, Nico : Danke für eure Antworten und Mühen, uns das ganze näher zu bringen.

    @Florian: “Das ist die Messung der Polarisation der Hintergrundstrahlung …”

    Okay, das habe ich ja verstanden, aber warum sind die (nur) am Rand zu sehen? Die müssten doch überall verteilt zu sehen sein – so schwarz zwischen rot und orange usw.- ?

  92. #92 Florian Freistetter
    18. März 2014

    @mmaker: “aber warum sind die (nur) am Rand zu sehen? “

    ?? Was soll da nur am Rand zu sehen sein? Die ist überall zu sehen. Hier sieht man doch die Messungen mitten im Bild: https://derstandard.at/plink/1395056871498?_pid=36522318#pid36522318

  93. #93 mmaker
    18. März 2014

    @Florian: ” Was soll da nur am Rand zu sehen sein? ”
    Da
    https://images.derstandard.at/2014/03/18/1395060665185-urknall880.jpg
    sieht man am Rand die schwarzen Bereiche. Soviel ich verstanden habe, sind das die gemessenen Wellen.
    Sie sprechen, denke ich, von den “Magnetpfeilen”, die auf dem Bild verteilt sind und polarisationen und so darstellen.

  94. #94 Florian Freistetter
    18. März 2014

    @mmaker: “sieht man am Rand die schwarzen Bereiche. Soviel ich verstanden habe, sind das die gemessenen Wellen.”

    Ok, ich verstehe worum es geht. Es GIBT kein “Bild” der Gravitationswellen. Die “Magnetpfeile” SIND die Messung der Wellen. Gravitationswellen kann man nicht sehen oder fotografieren. Aber wie ich in meinem Artikel ausführlich erklärt habe, können sie die Polarisation des Lichts beeinflussen und ihr bestimmte Muster aufprägen. Und genau dieses Muster zeigen die “Magnetpfeile”. Das Zeug am Rand ist einfach nur “Rauschen”; das sind die Grenzen des detektierten Gebiets am Himmel; da wo die Daten schlecht sind. Das hat keine physikalische Bedeutung.

  95. #95 Alderamin
    18. März 2014

    @McPomm, #70

    Kann man das vielleicht so zusammenfassen?
    Alan Guth hat in seiner Inflationstheorie u.a. vorhergesagt, dass dabei Gravitationswellen entstehen und diese das Licht (das erste sich frei ausbreitende Licht, das ist die kosmische Hintergrundstrahlung) B-modal polarisiert. Das BICEP-2-Team hat sich gesagt, dass sich diese Polarisierung im Muster der minimalen Helligkeitsschwankungen der Hintergrundstrahlung in ganz bestimmten wirbelförmigen Mustern niederschlagen müsste. Und genau diese Muster wurden jetzt gefunden. Oder?

    Nicht ganz. Alan Guth (und Andrei Linde) haben vmit der Inflationstheorie vorausgesagt, dass winzige Strukturen der Raumzeit durch die Inflation zu gewaltiger Größe aufgeblasen würden.

    Wenn für die Raumzeit dasselbe gilt, wie aus der Quantenmechanik für Teilchen und Energien bekannt, dann sollte die Raumzeit im allerkleinsten von zufälligen Dichtefluktuationen durchzogen sein, von winzigen Gravitatonswellen. Die Inflation würde diese riesengroß werden lassen, und wenn Photonen durch diese dermaßen vergrößerten Wellen laufen, würden sie zirkular polarisiert werden. Nach solchen Polarisationsmustern hat man gesucht und sie gefunden.

    Daraus folgt:
    – die Inflationstheorie von Guth und Linde stimmt anscheinend
    – für die Raumzeit gelten offenbar quantenmechanische Gesetze – ein Hinweis auf die bisher noch nicht gefundene Theorie der Quantengravitation
    – es gibt definitiv Gravitationswellen

  96. #96 Florian Freistetter
    18. März 2014
  97. #97 Markus Pössel
    Heidelberg
    18. März 2014

    @Florian: Gute Frage, ob das so wie bei der ESO sein könnte; wenn ich nochmal nachgoogle finde ich auf https://pole.uchicago.edu/spt/ aber:

    “The South Pole Telescope is a 10 meter diameter telescope operating at the NSF South Pole research station.”

    Also: Stationsname “NSF South Pole research station” und das South Pole Telescope ist wirklich (nur) die 10 Meter-Schüssel.

  98. #98 nautilu
    18. März 2014

    Mit dem Urknall kommt die Kirche vielleicht klar. Aber auch mit dem Multiversum?

    Also ich denke dass der Vatikan keine Probleme damit hätte dass es Päpste in anderen Universen gibt 🙂

    Die haben doch jetzt auch schon zwei und das ohne Konflikte.
    Von “Wir sind Papst” bis zu “Jeder ist Papst” ist es nur ein kleiner Schritt 🙂

  99. #99 Alderamin
    18. März 2014

    @myself, #30

    Hab’ den Artikel gefunden, “Back to the Big Bang”, Oktober 2013. Warum polarisieren Gravitationswellen die Strahlung, und wieso zirkular?

    Wenn unpolarisierte Photonen an einem Elektron gestreut werden, werden sie polarisiert. Angenommen, ein Bündel unpolarisierter Photonen kommt von links und wird an einem Elektron auf den Beobachter zu gestreut. Vor dem Streuen schwingen die Photonen also zufällig in allen Richtungen senkrecht zur Bewegungsrichtung von links nach rechts (d.h. in der Ebene, die aus den Richtunge auf/ab und vor/zurück aufgespannt wird). Nach der Streuung, wenn sie sich auf den Beobachter zu bewegen, fällt die vor/zurück-Komponente weg, weil diese jetzt in Bewegungsrichtung liegt, und nur die auf/ab-Komponente bleibt übrig, d.h. die Photonen haben jetzt alle eine senkrechte Polarisation.

    Für Photonen, die von oben auf das Elektron zu fliegen (Schwingungsebene links/rechts – vor/zurück) und zum Beobachter hin gestreut werden, bleibt demgemäß nur die waagerechte Polarisationsrichtung (links/rechts) erhalten.

    Angenommen, ein solches Elektron wird gerade von einer Gravitationswelle getroffen. Diese staucht den Raum in einer Richtung und streckt ihn senkrecht dazu. Nehmen wir an, die Welle streckt den Raum in Richtung links/rechts, staucht ihn inRichtung auf/ab. Das Elektron sieht dann die die Photonen, die von links und rechts kommen, zu höheren Wellenlängen (geringeren Energien) hin dopplerverschoben, die Photonen, die von oben und unten kommen, zu geringeren Wellenlängen (höheren Energien) dopplerverschoben. Am Elektron werden einige Photonen zum Beobachter hin gestreut. Die kühleren Photonen (aus der Ursprungsrichtung links/rechts) sind mit obiger Erklärung senkrecht polarisiert, die heißeren Photonen (aus der Ursprungsriuchtung auf/ab) sind waagerecht polarisiert. Da die heißeren Photonen mehr Energie haben, setzt sich ihr Muster durch und man beobachtet eine insgesamt etwas stärkere waagerechte Polarisationskomponente in der Überlagerung.

    Wenn die Gravitationswelle über das Elektron läuft, vertauschen sich die Richtungen von Streckung und Stauchung immer wieder. Mal ist die Streckung links/rechts und die Stauchung auf/ab, dann umgekehrt die Streckung auf/ab und die Stauchung links/rechts, usw. in einem oszillierenden Muster. Entsprechend verdreht sich auch laufend die Polarisationsrichtung des Lichts, das der Beobachter sieht.

    Nun sind die Gravitationswellen in der Hintergrundstrahlung riesengroß aufgebläht, und man beobachtet nicht, wie sich das Polarisationsmuster ständig ändert, sondern man sieht sozusagen eingefrorene Wellen, die an manchen Orten des Himmels die Stauchung in der einen Richtung und an anderen Orten die Stauchung in einer anderen Richtung verursachen. Und so entstehen die B-Moden der Polarisation, die dann am Himmel wie Wirbel aussehen.

    Schöner und mit Bildchen ist das im Artikel erklärt, aber das Heft ist kostenpflichtig.

  100. #100 Alderamin
    18. März 2014

    @J. Mischke, #73

    Wenn ALLE Materie das Universums bereits vorhanden war ( eben auf einem kleinen Punkt konzentriert ) muss doch eine unwahrscheinliche Gravitation geherrscht haben.

    Nein, während der Inflation war die Situation anders. Am Anfang war nur sehr wenig Masse (damals noch in Form von Energie, es gab noch keine Teilchen) vorhanden, es existierte ein winziger Raumbereich mit unglaublich hoher Vakuumenergie. Dieser Raumbereich wuchs, und damit vergrößerte sich die Gesamtenergie im eingeschlossenen Raum entsprechend, weil jedes Volumenelement neu hinzu kommenden Vakuums seinen Teil Vakuumenergie beisteuerte. Es entstand gewissermaßen Energie aus dem Nichts. Manche sagen: “auf Pump”. Und zwar von der Gravitation geliehen. Denn wandelte man die Vakuumenergie in Teilchen um und ließe sie unter ihrer Schwerkraft kollabieren, bekäme man die Energie wieder zurück. Sie steckt als potenzielle Energie in der Distanz der Volumenelemente.

    So wuchs also der Raum während der Inflationsphase gewaltig an, bis sich schließlich der Zustand des Vakuums zu einem geringeren Energiegehalt wandelte (Phasenübergang) und die vom Vakuum nicht mehr gehaltene Energie freigesetzt wurde. Aus ihr kondensierten dann Teilchen und Strahlung, und aus den Teilchen wurden (nach Vernichtung von Materie und Antimaterie und weiterer Abkühlung durch die fortgesetzte, aber nun verlangsamte Expansion des Alls) Wasserstoff und Helium (sowie ein bisschen Deuterium und Lithium).

    Die Materie des Universums war also erst nach dem Ende der Inflation freigesetzt, aber da war sie schon auf ein größeres Volumen verteilt und hatte so viel Schwung durch die Expansion mitbekommen, dass ihre Eigengravitation die Expansion nicht mehr aufhalten konnte. Den Punkt, in dem alle (heutige) Materie des Universums konzentriert war, hat es nach der Inflationstheorie nie gegeben.

  101. […] Inflation und damit zum Moment der Entstehung unseres Universums hat ScienceBlogger Florian hier bereits ausführlich berichtet. Diesen Bericht kann ich fachlich natürlich nicht ergänzen, möchte der werten […]

  102. #102 Astro#55
    München
    18. März 2014

    @Aldemarin, #100
    toll, verständlich und anschaulich erklärt. Für mich als Neuling in diesem Blog (habe seit gestern einige Beiträge gelesen) kann ich mir das Szenario bei der Entstehung der Inflation nun einigermaßen vorstellen.
    Eine brennende Frage habe ich dazu: Gilt bei der Transformation der Vakuumenergie in Masse die Einstein´sche Gleichung oder muss man annehmen, dass zu Beginn andere Naturgesetze vorhanden waren, welche die heutige Physik noch nicht kennt?

  103. #103 AP
    18. März 2014

    @Aldemarin, #100
    Ich habe das Greene Buch zweimal gelesen, wo er genau diesen Zusammenhang erklärt. Aber ich habe es nie so präzise verstanden, wie du es hier mit diesem Satz auf den Punkt gebracht hast:
    “Sie steckt als potenzielle Energie in der Distanz der Volumenelemente.”
    Danke!

  104. #104 Flanker
    Mannheim
    18. März 2014

    Ausgezeichneter Artikel, danke !

    Frage am Rande: dass Licht polarisiert sein kann, ist schon länger bekannt. Allerdings scheint es jetzt zwei Arten von Polarisation zu geben, die E-Moden und die B-Moden. Tatsächlich bezeichnet das eine den Richtungsvektor des elektrischen, der andere des magnetischen Feldes.

    Frage: ist das nun eine Eigenschaft jedes einzelnen Lichtquants, oder ist das ein Gruppenphänomen?

    Ist dann der Informationsgehalt des Lichts nicht wesentlich größer als bisher angenommen, wenn hier eine zweite Polarisation aufgeprägt ist? Und natürlich: wäre das nicht die ideale Art, mit der fremde Zivilisationen Kontakt mit uns aufnehmen könnten? Die warten, bis wir in der Lage sind, beide Orientierungen zu ermitteln. Was ja, wenn ich das richtig verstehe, eine brandneue Technik ist, die man nur teilweise beherrscht.

    Weshalb sich meine Enttäuschung auch in Grenzen hält, wenn es darauf noch keine Antwort gibt.

  105. #105 Alderamin
    18. März 2014

    @Astro#55

    Für die Energieumwandlung gilt Einstein, klar. Wie ich in #95 und Florian in seinem in #96 verlinkten Artikel erläutert haben, weist die Existenz der primordialen Graviationswellen zusätzlich noch auf eine Quantengravitation hin, für die wir noch keine annähernd vollständige Theorie haben und über die sich die Physiker seit ein paar Jahrzehnten die Köpfe zerbrechen.

    Was die inflationäre Expansionsphase selbst betrifft, auch dieser liegt Einstein zugrunde. Wieso expandiert der Raum überhaupt, was treibt ihn an? Es ist die Gravitation, und zwar in Form einer abstoßenden Kraft. Wie ist das möglich, wo Gravitation doch immer anziehend wirkt?

    Nach der ART hat nicht nur Masse eine Gravitationswirkung, sondern auch Druck. Wenn ich ein Medium komprimiere, wende ich Arbeit auf, die in das Medium übergeht (z.B. wird komprimierte Luft warm). Die zusätzliche Energie, die im Medium steckt, ist nach E=mc² mit einer Masse äquivalent, erzeugt also eine zusätzliche positive Gravitationskraft des Mediums.

    Wenn positiver Druck eine positive Gravitationswirkung hat, dann kann negativer Druck nach der ART eine negative, abstoßende Gravitationswirkung haben. Und die Annahme ist, dass das sogenannte “falsche Vakuum” während der Inflationsphase einen sehr großen, negativen Innendruck hatte (das klingt ein bisschen abgefahren, ein Vakuum hat Energie, ein Vakuum hat einen negativen Druck, aber ein physikalisches Vakuum ist alles andere als leerer Raum, lies mal bei “Hier wohnen Drachen” die Serie über das Vakuum).

    Aber wieso sorgt ein negativer Druck nicht für den Kollaps des Vakuums, sondern seine Expansion? Weil der negative Druck überall herrscht und überall gleich groß ist. Es gibt keinen Druckunterschied, der den Ausgleich sucht. Der ganze Raum ist im Druckgleichgewicht. Aber die abstoßende Gravitation ist da und es gibt nichts, keine Masse und keine freigesetzte Energie, das ihr entgegenwirken könnte, deswegen vergrößert sie das Vakuum hemmungslos. Bis das Vakuum irgendwo in einen Zustand geringerer Energie tunnelt und sich von dort aus in das gewöhnliche Vakuum unseres Weltraums verwandelt, wobei dann möglicherweise einige Naturkonstanten erst ihre Werte bekamen.

    Ich möchte Dir zu dem Thema das Buch von Lawrence Krauss, “Ein Universum aus Nichts: … und warum da trotzdem etwas ist” empfehlen, da wird dies ausführlich und verständlich erklärt.

  106. #106 Alderamin
    18. März 2014

    @Flanker

    Frage: ist das nun eine Eigenschaft jedes einzelnen Lichtquants, oder ist das ein Gruppenphänomen?

    Das elektrische Feld eines Photons hat irgendeine zufällige aber feste Richtung, die senkrecht zu seiner Bewegungsrichtung liegt. Von “polarisiertem Licht” kann man nur reden, wenn alle (oder die Mehrheit) der Photonen die gleiche Richtung des Felds aufweisen. Es ist also ein Gruppenphänomen.

    Frage am Rande: dass Licht polarisiert sein kann, ist schon länger bekannt. Allerdings scheint es jetzt zwei Arten von Polarisation zu geben, die E-Moden und die B-Moden. Tatsächlich bezeichnet das eine den Richtungsvektor des elektrischen, der andere des magnetischen Feldes.

    Es gibt linear und zirkular polarisiertes Licht. Bei zirkular polarisiertem Licht dreht sich die Richtung der Polarisation ständig, d.h. aufeinanderfolgende Photonen haben jeweils eine etwas weiter verdrehte Schwingungsrichtung.

    Die E- und B-Moden bezeichnen die Muster der Polarisation in der Hintergrundstrahlung, das hat soviel ich weiß nichts mit E-Feld und B-Feld zu tun (weiß jemand, woher die Bezeichnung kommt?). Wie die primordialen B-Moden zustande kommen, habe ich versucht, in #99 zu erklären.

    Ist dann der Informationsgehalt des Lichts nicht wesentlich größer als bisher angenommen, wenn hier eine zweite Polarisation aufgeprägt ist? Und natürlich: wäre das nicht die ideale Art, mit der fremde Zivilisationen Kontakt mit uns aufnehmen könnten? Die warten, bis wir in der Lage sind, beide Orientierungen zu ermitteln. Was ja, wenn ich das richtig verstehe, eine brandneue Technik ist, die man nur teilweise beherrscht.

    Nein, das ist weder neu noch aufregend. Polarisiertes Licht gibt’s in der Natur oft, z.B. Reflexion an leitenden Flächen, Streuung an kleinen Teilchen, an magnetisch ausgerichteten Staubteilchen etc. Zirkular polarisiertes Licht gibt’s seltener, man kann dies aber problemlos über eine Verzögerungsplatte aus linear polarisiertem Licht erzeugen, bei der ein Phasenunterschied zwischen zwei senkrechten Achsen erzeugt wird; wenn Du schon mal im 3D-Kino warst, hast Du sehr wahrscheinlich eine Brille mit Zirkularfiltern getragen (ein Auge links-, das andere rechtsdrehend zirkular polarisiert; das hat gegenüber linearer Polarisation den Vorteil, das man den Kopf auch mal zur Seite neigen darf oder nicht genau zentrisch vor der Leinwand sitzen muss, um beide Bilder sauber zu trennen).

    Wenn man anderen Zivilisationen ein auffälliges Signal schicken wollte, dann suchte man sich eher etwas, das in der Natur nicht so leicht erzeugt werden kann. Z.B. könnte man eine Folge von Primzahlen als Lichtpulse senden. Für Pirmzahlen gibt’s kein bekanntes natürliches Phänomen.

  107. #107 Astro#55
    18. März 2014

    @Alderamin, #105,

    danke für die ausführliche Erklärung und den Buchtip. In der Tat abgefahren… auch wenn ich das mit dem negativen Druck nur schwer verstehe. Die Vakuumenergie ist wirklich faszinierend und immerhin konnte sie schon experimentell über den Kasimir-Effekt nachgewiesen werde, korrekt?
    Das ist alles extrem spannend!

  108. #108 Alderamin
    18. März 2014

    @Astro#55

    Zum Casimir-Effekt, lies unbedingt mal diesen Artikel.

    Die Vakuumenergie ist eines der großen Rätsel der Physik. Wenn man ihren theoretischen Wert berechnet, so kommt man auf ein Ergebnis das um einen Faktor 10^120 zu groß ist – laut Lawrence Krauss die ungenaueste Vorhersage in der Geschichte der Physik.

    Die heutige, kleinere Vakuumenergie ist möglicherweise die Dunkle Energie, die das Weltall heute nach auseinander treibt, wenn auch ungleich weniger heftig, als ihre große Schwester, die kosmische Inflation. Aber wer behauptet, die Inflation sei Unsinn, sei an die kleine Schwester Dunkle Energie erinnert, deren Effekt wir live miterleben.

  109. #109 Christian
    Hannover
    19. März 2014

    Habe ich das richtig Verstanden: r=0.2 bedeutet, dass Anteil der Gravitationswellen an der Polarisation zu dem von Dichteschwankungen 1/5 beträgt, deshalb die Graviationswellen auf ca 1/100 der Planckenergie basieren und diese nur in der Inflationsphase und danach nicht mehr vorlag?

  110. #110 J. Mischke
    19. März 2014

    @100 alderamin.
    Danke für die Erklärung, werde ich mir noch mal in aller Ruihe anlesen.

  111. #111 Christian der 1.
    20. März 2014

    @Florian
    Wir lange brauchst du eigentlich für so einen Artikel bis der steht. ER ist ja doch umfangreich, bebildert, enthält links, ist fürs publikum nicht zu schwer verständlich, etc.
    gut als buchschreiber hast wahrscheinlich übung, aber wie du die artikel so im tagesrythmus raushaust ist schon erstaunlich

  112. #112 Florian Freistetter
    20. März 2014

    @Christian: Also diesen Artikel hab ich am Dienstag Vormittag geschrieben; hat ca 2 Stunden gedauert. Dazu kommen nochmal 1 bis 2 Stunden am Montag Abend, an denen ich mir diverse Artikel und Forschungsarbeiten zum Thema durchgelesen habe, damit ich auch weiß, um was es geht.

  113. #113 Christian der 1.
    21. März 2014

    würde mich interessieren was davon in 50, 100, 200 jahren noch bestand hat. und in welcher form.

    100 jahre wird wohl keiner von uns mehr erleben.

    die alten griechen (helliozentrisches weltbild), kepler, newton, einstein haben auch immer noch recht, wurden aber erweitert.

  114. […] Nachweis der kosmischen Inflation durch die Entdeckung von Gravitationswellen (bzw. deren Spuren) ist ja schon eine Weile her und wurde von Florian ausführlich erklärt (klickt […]

  115. #115 Hans Pörsch
    90480 Nürnberg
    6. April 2014

    Hier meldet sich H.P.Nbg. der überrascht war, als er persönlich aufgefordert wurde. Nun, denn, ich vermute Folgendes: Nach Prof.Lesch ISBN:378-3-442-15382-4 ist die Hubble’sche Expansion vergleichbar mit einem „hoch geworfenen Stein“. Und deswegen ist die gravitative Strukturierung des Universums derart, dass die Nobelpreis-träger 2011 höchstverdienlich für hier&heute “72% Dunkle Energie” und insgesamt “(4,6+23,4)% gravitative Materie” festgestellt haben. Für den Zeitpunkt der CMB-Erscheinung sollen es nach Wikipedia noch erst (10+15)% Dunkle Energie” und “(12+63)% gravitative Materie” gewesen sein. Die zeitliche Entwicklung der (Lage)Energetigkeit (=ĸ-ê-Kurve) folgte also der HubbleParameter-Kurve. Die (Lage)Energetigkeit des „geworfenen Steines“ zeichnet sich also ab in der gravitativen Strukturierung für hier&heute noch “28%” (Beharr)Energetigkeit (=Trägheit der bewegten Materie) und “72%” (Lage)Energetigkeit (=Höhe der gelagerten Materie); beides ergibt komplementär zusam-mengezählt “100%”. Die Gravitationswellen zeichneten eine Musterung namens „gravitative Strukturierung“ in Hubble′sche Expansionsblase ein. Und, an dem Verlauf der HubbleParameter-Kurve kann man rückwärts vermuten, dass die “25%” (Lage)Energetigkeit für die CMB-Erscheinung durch die Gravitationswellen eingeschrieben worden seien. Dann lässt sich im doppeltlogarithmischen Koordinatensystem anhand der ĸ-ê-Kurve noch weiter rückverfolgen, wann der «Inflations»„peak“ beendet war und die «Expansion» begann. {“Dunkle Energie” ist nichts Anderes als (Lage)Energetigkeit der Gravitationswelle (=HubbleParameter-Kurve)}.

  116. #116 Steffmann
    6. April 2014

    Uihh, ein Religionstroll, der sich in Physik probiert.

    https://www.hpoersch-41stein.de/42378.html

    Öfter mal was Neues.

  117. #117 Adent
    6. April 2014

    @Hans Pösch
    Form und Inhalt?

  118. #118 PDP10
    6. April 2014

    @Adent:

    Ich dachte schon, ich wäre der einzige der das für den Output eines Nonsensetext-Generators hält …

  119. #119 Kryptonoob
    7. April 2014

    OMG, Hans#115!!!
    (Lage)Energetigkeit…soll er damit pot. Energie meinen?
    Und (Beharr)Energetigkeit kin. Energie?
    Oder irgendwas ätherisches?
    Ist für mich echt der Brüller des Tages 🙂

  120. #120 max
    5. Mai 2014

    Den Urknall hat es nicht gegeben, genausowenig die Inflation. Ebenso gibt es keine dunkle Materie und dunkle Energie. Alles nur erfundene Absurditäten, um die Urknalltheorie noch irgendiwe halten zu können.
    Die Rotverschiebung der entfernten Galaxien kommt nicht vom Dopplereffekt, die Gravitationswellen nicht von der Inflation. Dafür gibt es Theorien, die millionenfach plausibler sind.
    Die z.Zt. wohl beste Theorie ist WSM von Milo Wolff.
    Da weiß man dann auch gleich was Masse, Gravitation, Bewegung und Zeit ist, und wie sie funktionieren.

    Wer heute noch an den Urknall glaubt kann auch gleich an die Schöpfungsgeschichte glauben, und an den Osterhasen 🙂

  121. #121 max
    5. Mai 2014

    ach ja, ich sollte wohl doch noch etwas nähere Informationen zu meiner oben getätigten Aussage hinzufügen.

    Hier ein internationaler Protest von Wissenschaftern gegen die Urknalltheorie:
    https://cosmologystatement.org/

    Hier etwas zum Lesen auf deutsch:
    https://hauptplatz.unipohl.de/Wissenschaft/SackgasseUrknall.htm

    und Sorry! für den Osterhasen 😉

  122. #122 Florian Freistetter
    5. Mai 2014

    @max: “Den Urknall hat es nicht gegeben, genausowenig die Inflation. Ebenso gibt es keine dunkle Materie und dunkle Energie. Alles nur erfundene Absurditäten, um die Urknalltheorie noch irgendiwe halten zu können.”

    Ok. Entweder ist das ganze hier Satire oder du gehörts leider auch zu denen, die auf die Vorurteile der Pseudowissenschaftler reingefallen sind. Ich würde dir ja gerne erklären, was der Unterschied zwischen “Beobachtung” und “Erfindung” ist. Aber ich fürchte, das willst du gar nicht wissen, oder? Über dunkle Materie könntest du dich zB hier informieren: https://scienceblogs.de/astrodicticum-simplex/?p=10955
    Über den Urknall zB hier: https://scienceblogs.de/astrodicticum-simplex/2011/02/24/den-urknall-gab-es-wirklich-teil-2-das-licht-aus-der-vergangenheit/

  123. #123 max
    5. Mai 2014

    “Vorurteile der Pseudowissenschaftler”?
    wie z.B. Newton, Einstein, Hubble usw.?

    Der Theologe, Priester und Astrophysiker Lemaitre hat den Urknall bereits 2 Jahre vor der von Hubble entdeckten Rotverschiebung erfunden und sich damit schließlich großes Wohlwollen von Papst Pius gesichert:
    “Auf einer Tagung im November 1951 akzeptierte die Päpstliche Akademie der Wissenschaften Lemaîtres Theorie. Papst Pius XII. führte in einem abschließenden Vortrag aus, der mit dem Urknall zeitlich festlegbare Anfang der Welt sei einem göttlichen Schöpfungsakt entsprungen.”

    Interessant, dass der Physiker Edwin Hubble die Rotverschiebung niemals als eine Flucht der Galaxien und als eine Expansion des gesamten Alls akzeptiert hat! (siehe Spektrum der Wissenschaft vom Sept. 1993, Seite 82).
    Die ohne gesicherten Grundlagen erfundene Urknalltheorie des Priester-Astrophysikers hat sich dennoch durchgesetzt. Wohl auf Grund der “In God we trust”- Universitäten und der Verteilung der Forschungsgelder.
    Seither wird diese Theorie mit viel Geld gegen alle widerlegenden Beobachtungen und mit immer neuen Absurditäten gestützt. Hoffentlich dauert das nicht wieder so lange wie mit dem geozentrischen Weltbild…

    Über die dunkle Materie und den Urknall bin ich umfassend informiert, und die Bibel habe ich auch gelesen 🙂

    Und glaubst du wirklich, dass das hier Satire ist?:
    https://cosmologystatement.org/

  124. #124 Kallewirsch
    5. Mai 2014

    > “Vorurteile der Pseudowissenschaftler”?
    > wie z.B. Newton, Einstein, Hubble usw.?

    Wenn wir über moderne Kosmologie reden, hat es wohl wenig Sinn, anerkannte Wissenschaftler aus der Vergangenheit zu zitieren.
    Statt dessen musst du schon Leute heranziehen, die in den 70- er oder 80-er Jahren, bzw. noch später, den Zenit ihrer wissenschaftlichen Karriere erreicht haben.

    Aristoteles war zu seiner Zeit eine anerkannte Autorität. Nichts desto trotz waren seine Ideen zur Fallbeschleunigung von Körpern einfach nur grundfalsch. Selbiges hier. Weder Newton, noch Einstein, noch Hubble kannten weder die Beobachtungen bzw. Messungen noch die Weiterentwicklungen des theoretischen Unterbaus. Ist ein Beleg durch Autorität grundsätzlich schon problematisch, so taugt er umso weniger, je weniger die jeweilige Autorität mit den modernen Messergebnissen vertraut war und je größer der zeitliche Abstand zu diesen ist. Weder Newton, noch Einstein oder Hubble wussten zb etwas von der Existenz der kosmischen Hintergrundstrahlung.

  125. #125 Kallewirsch
    5. Mai 2014

    Hier ein internationaler Protest von Wissenschaftern gegen die Urknalltheorie:
    https://cosmologystatement.org/

    Du mögest bitte etwas genauer lesen, was der Sinn der Petetion ist. Da geht es nicht um einen Protest gegen den Big Bang.
    Sondern es geht darum, dass es einigen Leuten stinkt, dass sie keine Förderung von Forschungsprojekten gibt (bzw. ihrer Ansicht nach massiv zu wenig), die sich zum Ziel setzen, die Big Bang Theorie weiteren Prüfungen zu unterziehen. Das da unter Umständen rauskommt, dass Big Bang so nicht stimmt bzw. modifiziert werden muss, mag ein Grund sein, warum diese Forschungsförderung nicht statt findet. Der Protest richtet sich in erster Linie nicht gegen Big Bang an sich, sondern es dreht sich um die Unterfinzanzierung von Forschungsprojekten.

  126. #126 max
    5. Mai 2014

    @Kallewirsch
    ad1: mir geht es dabei um den Begriff “Pseudowissenschaftler” für Wissenschaftler die nicht mit dem Mainstream arbeiten, bzw, dessen Theorien nicht zu untermauern versuchen. Es geht mir nicht um deren (späteren) “Autorität”.

    ad2: ich lese den Artikel doch etwas anders:
    Even observations are now interpreted through this biased filter, judged right or wrong depending on whether or not they support the big bang. So discordant data on red shifts, lithium and helium abundances, and galaxy distribution, among other topics, are ignored or ridiculed. This reflects a growing dogmatic mindset that is alien to the spirit of free scientific inquiry.
    Der Protest richtet sich durchaus auch gegen die BB Theory und besonders deren Erhebung zur Quasi-Religion trotz der vielen widersprüchlichen Beobachtungen und der für die Erhaltung der Theory eingeführten Paradoxien.

  127. #127 Alderamin
    5. Mai 2014

    @max

    Und glaubst du wirklich, dass das hier Satire ist?

    Nö, aber auch schon wieder 10 Jahre alt, seitdem hat sich in Form von WMAP und PLANCK eine Menge getan. Natürlich gibt’s Anhänger aller möglichen abstrusen Theorien, und wer sein Leben lang von seiner eigenen alternativen Erklärung überzeugt war, der ändert nicht mal so eben seine Meinung wegen ein paar Fakten, die dieser widersprechen.

    Das Problem bei den alternativen Theorien ist, dass sie ganz wesentliche Beobachtungen nicht erklären können. Z.B. ist völlig unklar, woher die Materie stammt/ständig nachgeliefert wird, und wie dabei das beobachtete Verhältnis der Elemente Wasserstoff/Deuterium-Helium-Lithium herauskommt, das sich während der Fusionsphase im Feuerball des Urknall von ganz zwanglos ergibt. Außerdem können sie die Feinstruktur der Hintergrundstrahlung nicht erklären – dass anfangs eine solche nicht gemessen worden war, hat Eric Lerner ja noch als Argument für seine Plasma-Kosmologie angeführt. Nur haben wir die Feinstruktur mittlerweile so gut gemessen, dass sich darin sogar der Einfluss der Inflation zeigt (siehe Artikel oben). Schon gar nicht erklärt das Plasma-Universum die räumliche Korrelation in der Hintergrundstrahlung, die berühmte Multipol-Kurve.

    Die Beweislage gegen alternative Kosmologien ist mittlerweile erdrückend, die Ergebnisse der Messungen der Expansion des Universums, der Feinstruktur der Hintergrundstrahlung und der Verteilung Dunkler Materie stützen das Lambda-CDM-Modell und bestätigen sich gegenseitig.

    Und was ist an der Dunklen Materie so unglaubwürdig? Neutrinos sind auch Dunkle Materie, warum sollte es nicht noch mehr Teilchen geben, die mit normaler Materie kaum wechselwirken, und die wir deshalb nur sehr schwierig nachweisen können? Wenn sie nur über Gravitation wechselwirken, können unsere irdischen Experimente sich daran totmessen ohne etwas zu finden. Hingegen kann man die Verteilung der DM in Galaxienhaufen über Messungen der Lichtablenkung dahinter liegender Galaxien wunderbar nachweisen.

    Und die beschleunigte Expansion des Universums ist ein Fakt, das eindeutig aus den Messungen der Entfernung von Supernovae hervorgeht. Die “Dunkle Energie” ist nur ein Name für diese Beschleunigung. Ihre Ursache ist noch unbekannt.

    Viel abstruser ist die Annahme, dass irgendwo Materie aus dem nichts entstünde und zu höheren Kernladungszahlen fusioniert, ohne dass wir davon etwas mitbekommen (Gammastrahlung z.B.). Aus den Sternen kommen die Elemente jedenfalls nicht – denn wo ist im primordialen Gas das Eisen, mit dem die Sterne in der Milchstraße das interstellare Gas anreichern? Wo Kohlenstoff, Stickstoff und Sauerstoff, die beim CNO-Zyklus anfallen? Wo sind die Sterne, die Billionen Jahre alt sind (Rote Zwerge werden so alt), wo die Sternen- und Galaxienreste eines unendlichen Steady-State-Universums? Was ist an der permanenten Entstehung von Materie aus dem Nichts plausibler als am Urknall, der wenigstens den Energieerhaltungssatz wahren kann (siehe dazu Lawrence Krauss, Ein Universum aus dem Nichts).

    Über die Unzulänglichkeiten der Plasma-Kosmologie:
    https://en.wikipedia.org/wiki/Plasma_cosmology#Comparison_to_mainstream_astrophysics

  128. #128 Alderamin
    5. Mai 2014

    @max

    Der Protest richtet sich durchaus auch gegen die BB Theory […] trotz der vielen widersprüchlichen Beobachtungen und der für die Erhaltung der Theory eingeführten Paradoxien.

    Welche sind das denn, zum Beispiel?

  129. #129 Florian Freistetter
    5. Mai 2014

    @max: Mit”Pseudowissenschaftlern” meinte ich die Vorstellung, die Wissenschaftler hätten dunkle Materie oder den Urknall nur “erfunden” um irgendwelche “Theorien retten” zu können. Wer so etwas sagt, der zeigt, dass er nicht wirklich viel Ahnung davon hat, wie das in der Wissenschaft tatsächlich abgelaufen ist. Und auch die Geschichte von Lemaitre, der den Urknall zu “Gottes Gnaden” entwickelt hat, hat nicht viel mit der Realität zu tun; genau so wenig wie das Vorurteil, man dürfe in der Wissenschaft nur das erforschen, was irgendein ominöser “Mainstream” vorgibt. Du kannst ja gerne mal die aktuell im Blog laufende Serie (bzw. das Buch auf der sie basiert) lesen: https://scienceblogs.de/astrodicticum-simplex/2014/05/05/die-perfekte-theorie-8-schwarze-loecher-sind-real/

    Ich weiß ja, dass es viel aufregender zu sein scheint, dagegen zu sein. Da kann man dann den ganzen Kram von wegen Elite, Märtyrer usw abziehen. Aber die echte Welt ist viel aufregender als all das. Man muss nur mutig genug sein, sich mal ernsthaft damit auseinander zu setzen…

  130. #130 Florian Freistetter
    5. Mai 2014

    P.S. Nur mal so gefragt: Die ganzen wissenschaftlichen Messungen von wegen Elementhäufigkeit, Spektrum der Hintergrundstrahlung, Gravitationskraft oder eben die hier im Artikel angesprochene Polarisation und all die anderen Vorhersagen der LCDM-Kosmologie die durch Beobachtungen bestätigt worden sind: Sind das alles gefälschte Daten oder wie ist das abgelaufen?

  131. #131 max
    6. Mai 2014

    post#127

    >”Natürlich gibt’s Anhänger aller möglichen abstrusen Theorien, und wer sein Leben lang von seiner eigenen alternativen Erklärung überzeugt war, der ändert nicht mal so eben seine Meinung wegen ein paar Fakten, die dieser widersprechen.”Das Problem bei den alternativen Theorien ist, dass sie ganz wesentliche Beobachtungen nicht erklären können.Und die beschleunigte Expansion des Universums ist ein Fakt, das eindeutig aus den Messungen der Entfernung von Supernovae hervorgeht.<

    Eben nicht. z.B.:
    https://www.livescience.com/33522-accelerating-universe-dark-energy-illusion.html
    Aber selbst die gesamte Rotverschiebung lässt sich viel plausibler und passender zu den gemessenen Werten erklären. z.B.:
    https://articles.adsabs.harvard.edu/full/1993ApJ…405…51N
    oder:
    https://redshift.vif.com/JournalFiles/V13NO2PDF/V13N2HAR.pdf

    Das hier ist ein sehr komplexes Thema, und ich kann schon vom Zeitaufwand nicht soviel dazu schreiben. Daher kann ich nur bitten, die verlinkten Artikel aufmersam zu studieren. Ich wähle die Links so gut ich in der Eile kann aus. Ich möchte daher eine Webseite (und die darin enthaltenen Unterlinks) auch dem geneigten Mitleser empfehlen, und hoffe, dass darin auch einige weitere Fragen des post#127 beantwortet sind.
    https://www.physikgrundlagen.de/universum.html#7

    Ich habe nicht nur die Standard Theorien und etwa die Arbeiten von Eric Lerner sondern auch viele andere gelesen. z.Zt. ist für mich wie gesagt die WSM Theory von Milo Wolff am verfolgenswertesten. Wer sich einmal das Gehirn durchblasen will, alt aber gut: https://www.amazon.com/Exploring-Physics-Universe-Adventurers-Guide/dp/0962778702/ref=la_B001KCBD3K_1_2?s=books&ie=UTF8&qid=1399332370&sr=1-2

    Welche Theory auch immer sich in Zukunft als am richtigsten heraustellen wird, die Urknall Theory wird es bestimmt nicht sein. Bin ich mir sicher 🙂

  132. #132 max
    6. Mai 2014

    post#128
    Das ist nur die Spitze des Eisbergs an Paradoxien:
    https://www.circlon.com/living-universe/054-The-Big-Bang-Paradoxes.html

    aber für mich geeignet, die Urknall Theory wie eine heiße Kartoffel fallen zu lassen 🙂

  133. #133 max
    6. Mai 2014

    post#129
    Den Begriff “Pseudowissenschaftler” hast du in die Diskussion gebracht:
    >…oder du gehörts leider auch zu denen, die auf die Vorurteile der Pseudowissenschaftler reingefallen sind.”

    Ich denke, es ist klar, dass ich das nicht so stehen lassen kann… 😉

  134. #134 max
    6. Mai 2014

    post#130
    Die Messungen sind nicht gefälscht, aber bei weitem nicht so unumstößlich genau wie das suggeriert wird. Weiters werden alle Ergebnisse die dem Urknall dienen können gerne mit weiteren Variablen, Konstanten und Theorien so hingebogen, dass sie passen können. Dem Urknall widersprechende Ergebnisse werden einfach ignoriert. Insgesamt ergibt sich dadurch ein enormes Feld für Missinterpretation.

    Dafür noch eines der letzten Zitate von Einstein:
    „Wenn Ihr Euch nicht bessert, soll Euch doch alle der Teufel holen!“
    🙂

  135. #135 max
    6. Mai 2014

    da der Beginn meines Post#131 weiter oben verstümmelt ist, hier nocheinmal der Anfang:

    post#127
    “Natürlich gibt’s Anhänger aller möglichen abstrusen Theorien, und wer sein Leben lang von seiner eigenen alternativen Erklärung überzeugt war, der ändert nicht mal so eben seine Meinung wegen ein paar Fakten, die dieser widersprechen.”

    Genau!!
    Ich bin früher ebenfalls mit der Uknalltheorie mitgeschwommen, aber inzwischen “geheilt”. 🙂
    Wie wohl manch andere auch:
    St. Hawking in „Eine kurze Geschichte der Zeit“, Seite 73:
    „Inzwischen habe ich meine Meinung geändert und versuche nun, die Physiker davon zu überzeugen, dass das Universum nicht mit einer Urknall-Singularität entstanden ist!

    ”Das Problem bei den alternativen Theorien ist, dass sie ganz wesentliche Beobachtungen nicht erklären können.”

    Das Problem mit den alternativen Theorien ist, dass es kaum umfassende gibt. Wie denn auch, praktisch sämtliche Forschungsgelder für die Kosmologie fließen in den Urknall-Topf. Alternative Theorien müssen von den Wissenschaftlern praktisch aleine und in ihrer Freizeit erarbeitet werden. Mäzene gibt es heute auch nicht mehr. Trotzdem sind deren Ergebnisse so signifikant, dass die Urknall Theory wohl nicht mehr lange gehalten werden kann.

    Expansion des Universums und Rest: siehe oben

  136. #136 max
    6. Mai 2014

    oje, ein Link oben #131 funktioniert nicht:

    Aber selbst die gesamte Rotverschiebung lässt sich viel plausibler und passender zu den gemessenen Werten erklären. z.B.:
    https://articles.adsabs.harvard.edu/full/1993ApJ…405…51N
    (https://articles.adsabs.harvard.edu/full/1993ApJ…405…51N)

  137. #137 Adent
    6. Mai 2014

    @max
    #131
    Und für dich ist es also viel wahrscheinlicher, daß der liebe Herr Carter das Ganze als Buch veröffentlicht (ohne irgendwelches Review), weil er von der wissenschaftlichen Community so ganz pöhse unterdrückt wird (passiert ja dauernd sowas, sehe ich jeden Tag, Achtung Ironie!). Deshalb nimmt er dann lieber die Kohle für ein Buch, statt einen gescheiten Artikel über seine Theorie in einem peer reviewed Journal zu veröffentlichen (was andere Wissenschaftler das eine oder andere mal in ihrer Karriere durchaus zustande bringen), oder habe ich das falsch verstanden?
    Kennst du den Fachbegriff für diese Art Vorgehen/Behauptungen (unschuldig guck)?

  138. #138 Florian Freistetter
    6. Mai 2014

    @max: “Weiters werden alle Ergebnisse die dem Urknall dienen können gerne mit weiteren Variablen, Konstanten und Theorien so hingebogen, dass sie passen können. Dem Urknall widersprechende Ergebnisse werden einfach ignoriert. “

    Na du weisst ja ganz exakt, wie es in der Welt der Wissenschaft zu geht. Mit so einem superklugen Alleschecker muss ich dann gar nicht weiter diskutieren. Du weisst ja sowieso schon alles und hast erklärt, dass dich kein Argument von deiner Meinung abbringen kann. Also wünsche ich dir noch viel Spaß dabei, die Urknall-Theorie doof zu finde (es ist halt nur schade, dass du keine wirkliche Ahnung von echter Kosmologie hast sondern dein verzerrtes “Anti-Mainstream-Wissen” – die echte Kosmologie ist enorm spannend).

  139. #139 Alderamin
    6. Mai 2014

    @max #131

    Dann wollen wir mal…

    “Time Paradox”:
    – die CMB-Strahlung hat einen Rotverschiebungsfaktor von 1080, nicht 2000 (wenn man schon gegen den Urknall argumentiert, sollte man wenigstens die richtigen Zahlen wissen). Das entspricht bei gemessenen 2,7 K (nicht “°”) einer ursprünglichen Temperatur von nahezu 3000 K. Bei dieser Temperatur wurde das ursprüngliche Plasma neutral und durchsichtig.

    – Die Rotverschiebung ist alles andere als linear über die Entfernung, weil die Rotverschiebung logischerweise umgekehrt proportional zur Größe des Weltalls ist: Wellenlänge doppelt so lang -> Universum damals halb so groß (die korrekte Formel lautet a=1/(1+z) mit dem Skalenfaktor a und der Rotverschiebung z). Bei einer Rotverschiebung von 1000 war das Weltall 1/1000 so groß wie heute, die CMB-Strahlung stammt aus einem jungen Universum von 380000 Jahren. Zur Zeit der ersten Quasare bei (wenn’s stimmt) 12 Milliarden Jahren Lichtlaufzeit war das Weltall schon tausende Mal älter, fast 10% des heutigen Alters, daher Rotverschiebungen von um die 10. Dazwischen leuchtete nichts, was wir heute noch sehen könnten (mWn sind die ältesten Galaxien sind bei 500 Millionen Jahren Weltalter nachgewiesen; Einzelsterne sieht man auf diese Entfernung nicht mehr).

    “Space Paradox”:

    The shape of starlight’s wavelength distribution curve is the same as the blackbody curve, but its intensity is thousands of time less than the blackbody radiation for its wavelength curve.

    – Unsinn, Sterne haben, bis auf die Spektrallinien, ziemlich genau die Schwarzkörperstrahlung, die ihrer Temperatur entspricht, z.B. bei der Sonne von 5800 K liegt das Maximum im Grünen. Warum schreibt der Autor so was? Zeugt nicht von Ahnung. Auf den verlässt Du Dich?

    Cosmic rays also have a temperature of about 3°K

    – Ähem. Kosmische Strahlen sind Gamma- und Partikelstrahlung, die haben ein paar Millonen bis Milliarden K Temperatur. Er meint wohl die Hintergrundstrahlung, die ist aber eine nahezu perfekte Schwarzkörperstrahlung, kann man in Wikipedia nachlesen (ich verlinke hier nicht alles, mehr als 2 Links pro Post landen eh’ in der Mod).

    – Die folgende Erklärung mit der Energiedichte ist Quatsch. Wenn sich das Weltall den Radius verdoppelt, steigt der Skalenfaktor um den Faktor 2, die Wellenlänge ebenfalls, die Energie des Photons halbiert sich, die Temperatur sinkt auf die Hälfte, die Schwarzkörperkurve wird nach dem Wienschen Verschiebungsgesetz halb so lang. Die Gesamtenergie bleibt tatsächlich nicht erhalten (Orte mit verschiedenen z sind kein Inertialsystem).

    – Die Annahme, dass sich die Galaxien lokal nicht bewegen, ist nicht ad-hoc. Erstens war der Urknall keine Explosion, der den Galaxien irgendeinen “Schwung” gab, den sie heute noch haben, sondern schon da wuchs lediglich der Raum. Das tut er heute noch (nur viel langsamer), und zwar beschleunigt (mit “Schwung” würde er höchstens zunehmend langsamer wachsen können). Zweitens befinden wir uns relativ zur Hintergrundstrahlung beinahe in Ruhe (ca. 600 km/s aufgrund der Drehung der Galaxis und ihrer Eigenbewegung sind nicht viel). Wir müssten also zufällig genau im Zentrum des Weltalls sitzen, wenn die Rotverschiebung der Galaxien eine echte Bewegung wäre.

    – Dass gravitativ oder anderweitig gebundene Körper nicht mit expandieren verwundert eigentlich gar nicht so sehr, wenn man bedenkt, dass es keine Kraft braucht, einer konstanten Expansion das Gleichgewicht zu halten. Martin Bäker schrieb neulich darüber. Erst über größere Entfernungen, wo die Kräfte zwischen den Galaxien vernachlässigbar werden, kommt die Expansion zum Tragen.

    Man kann argumentieren dass die Expansion des Universums den Energieerhaltungssatz verletzt, oder man kann die Ansicht vertreten, dass die Energie in potentielle Energie übergeht, dann bleibe sie erhalten. Es gibt Anhänger beider Positionen. Als Alternative wird ja gelegentlich “Tired Light” genannt. Dieses verletzt die Energieerhaltung mindestens genau so, denn an E=h*f ändert sich ja nichts.

    Das wichtigste Argument gegen das “Space-Paradox” ist, dass man die Expansion des Weltalls eindeutig messen kann, indem man die Rotverschiebung der Galaxien in Beziehung zur Entfernung setzt, die wir mittlerweile mit weniger als 10% Fehler anhand von Supernovae messen können, und ja, mit zunehmender Entfernung wird die Rotverschiebung größer, das Licht röter und weniger energiereich. Und die Rotverschiebung der Hintergrundstrahlung passt genau zur Rekombinationstemperatur von Wasserstoff. Was soll man daran noch herumargumentieren? Das ist ein Messergebnis.

    “Antimatter-Paradox”
    – die Abwesenheit von Antimaterie wird mit Prozessen, die die CP-Symmetrie verletzen, erklärt. In der Tat ist noch nicht klar, wie die Baryogenese genau abgelaufen ist. Da ist noch Forschungsbedarf.
    – Wie erklären die Kritiker denn die Entstehung von Materie? Wenn das Weltall ewig wäre, dann müsste sie ja nachgeliefert werden. Wenn nur Materie ohne Antimaterie entstünde, dann bestünde das Paradox ja weiterhin. Wenn nicht, wo entsteht die Materie und wo ist die Antimaterie? Wo kommt diese Energie her?

    “Energy Paradox”
    – das Argument mit der Photonenergie hatten wir oben schon.
    – die Galaxien haben keine kinetische Energie aufgrund ihrer Expansion, allenfalls eine potenzielle aufgrund des Abstands. Ansonsten gäbe es ja auch keinen kosmischen Horizont, jenseits dessen die Expansion Lichtgeschwindigkeit übertrifft. Da keine Bewegung vorliegt, kann man auch keine kinetische Energie daraus ableiten und die Lichtgrenze besteht nicht.
    – das Weltall durchmisst nicht 13,78 Milliarden Lichtjahre, das ist lediglich die Lichtlaufzeit, die ein Photon, das der Expansion entgegen läuft, vom Horizont bis zur Erde braucht. Der Horizont liegt vielmehr bei einem Radius von 46 Milliarden Lichtjahren in sogenannter “Proper Distance” gemessen. Das Weltall ist ja hinter dem Photon weiter gewachsen, es hat nicht die gesamte endgültige Strecke zurücklegen müssen, sondern nur das Stück, das jeweils noch vor ihm lag und weiterhin expandierte.

    “2.7°K Cosmic Blackbody Radiation Paradox”
    – Energieerhaltung siehe oben
    – die relative Intensität der Strahlung der Milchstraße tut genau was zur Sache?
    – kosmische Strahlung ist Partikel- und Gammastrahlung, die hat mit der Hintergrundstrahlung nichts zu tun und kommt in der Tat überwiegend aus der Milchstraße (Pulsare, Supernovareste). Die tut hier auch nichts zur Sache (und hat keine 2,7 K).

    The Big Bang promoters view the 2.7°K temperature as a random point in time that began shortly after the Big Bang at a temperature of about 3000°K and has been slowly cooling down ever since

    – das ist kein “Random Point”, sondern die Temperatur, bei der sich das Plasma aus Elektronen und Protonen in neutrales Wasserstoffgas wandelte. Ein Plasma hat freie Elektronen die Photonen jeder beliebigen Energie absorbieren und streuen können, deshalb ist es undurchsichtig und leuchtend hell. Ein neutrales Gas kann nur einzelne Spektrallinien absorbieren und ist daher durchsichtig. Deswegen konnten erst nach der Abkühlung unter 3000 K die Photonen ungehindert geradeaus fliegen und uns heute erreichen.
    – ein solches Plasma hat eine nahezu perfekte Schwarzkörperstrahlungskurve bei 3000 K. Man schaue sich Sterne vergleichbarer Temperatur an, die haben das gleiche Plasma, die gleiche Strahlungskurve.

    The only significant variation that has been measured in the 2.7° CBR is its dipole anisotropy.

    – Unsinn, es gibt eine Feinstruktur mit der Multipolkurve, die winkelabhängige Korrelationen aufzeigt (die mögen die Kritiker bitte mal erklären), und Polarisation (siehe Artikel oben).
    – Ja, die Dipolanisotropie zeigt an, dass wir uns im Raum relativ zu einem Ort, der relativ zur Hintergrundstrahlung in Ruhe ist, bewegen, und zwar mit insgesamt 600 km/s. Darin enthalten ist die Bewegung der Sonne mit 19 km/s in der Milchstraße, ihr Umlauf um das Milchstraßenzentrum mit 250 km/s und die Bewegung der Milchstraße auf die Andromedagalaxie zu, mit der sie in 5 Milliarden Jahren kollidieren wird. Was ist jetzt daran paradox? In anderen Galaxienhaufen sehen wird doch ähnliche Geschwindigkeiten der Galaxien. So groß, dass es dunkle Materie braucht, damit es diese nicht zereisst. Ich verstehe das Argument nicht.

    “Hubble Constant Paradox”

    However, all that is really measured is a gradual increase in the wavelengths of photons over great periods of time or distance.

    – die Expansion des Universum erklärt genau diesen Effekt.
    – eine Eigenbewegung entfällt werden des oben genannten Arguments, dass wir ansonsten im Zentrum des Universums sitzen müssten
    – die Expansion passt genau zur Temperatur der Hintergrundstrahlung für den Übergang zum neutralen Gas
    – Gamma-Ray-Bursts und die Dauern von Supernova-Explosionen sind um den gleichen Faktor 1/(1+z) verlängert, der Dopplereffekt betrifft nicht nur Licht, sondern alle zeitlichen Vorgänge, was nur mit einer Expansion (oder echten Bewegung, die ausscheidet) zu erklären ist.
    – die Alternativerklärung Nr. 3 im Text (alte Photonen) löst auch nicht die Energieerhaltung und erklärt die Verlängerung von Supernovae nicht.

    Also, ich finde den Text nicht sehr überzeugend.

    @#127

    St. Hawking in „Eine kurze Geschichte der Zeit“, Seite 73:
    „Inzwischen habe ich meine Meinung geändert und versuche nun, die Physiker davon zu überzeugen, dass das Universum nicht mit einer Urknall-Singularität entstanden ist!

    Du weißt aber schon, dass Hawking ein Anhänger der Urknalltheorie ist? Ob das Weltall aus einer Singularität entstand, ist außerdem gar nicht unumstößlicher Bestandteil der Urknalltheorie, im Gegenteil, die Inflationstheorie benötigt nur ein Planck-Volumen mit hoher Energiedichte (falsches Vakuum) als Ausgangs”punkt”. Gemäß der String- und Quantengravitationstheorien gibt es ohnehin keine punktförmigen Singularitäten. Singularitäten bestehen nur da, wo man die Formeln der Relativitätstheorie in Bereichen anwendet, wo sind nicht experimentell überprüft ist und ohnehin der Quantentheorie widerspricht. Alle Physiker sind sich darüber im Klaren, dass in diesen Bereichen eine neue vereinheitlichende Theorie nötig ist. Überraschung, die Wissenschaft weiß noch nicht alles!

    Das Problem mit den alternativen Theorien ist, dass es kaum umfassende gibt. Wie denn auch, praktisch sämtliche Forschungsgelder für die Kosmologie fließen in den Urknall-Topf. Alternative Theorien müssen von den Wissenschaftlern praktisch aleine und in ihrer Freizeit erarbeitet werden. Mäzene gibt es heute auch nicht mehr. Trotzdem sind deren Ergebnisse so signifikant, dass die Urknall Theory wohl nicht mehr lange gehalten werden kann.

    Du verwechselst Ursache und Wirkung. Die Forschung zu alternativen Theorien ist nicht schlecht, weil sie kein Geld bekommt. Sie bekommt kein Geld, weil sie so schlecht ist. Weil sie nichts erklärt. Weil sich fast alle von ihr abgewendet haben. Weil sie niemanden mehr überzeugt. Das war zu Hoyles früheren Zeiten ja noch anders.

    Die Standardkosmologie wird auch nicht richtiger, weil sie mehr Geld bekommt. Wer auf sich aufmerksam machen will, der sucht nach Dingen, die noch nicht erklärt sind, so wird man bekannt. Man bekommt keinen Ruhm, wenn man Bekanntes nur bestätigt. Man forscht also mit dem vielen Geld also auch daran, wie man die Urknalltheorie zu Fall bringen kann. Wenn das so einfach wäre, hätte man das längst hinbekommen. Das Gegenteil ist passiert.

    @#135 Der Link geht leider nicht.

  140. #140 Alderamin
    6. Mai 2014

    @max

    Weiters werden alle Ergebnisse die dem Urknall dienen können gerne mit weiteren Variablen, Konstanten und Theorien so hingebogen, dass sie passen können.

    Wie beim Gravitationsgesetz: da wurde eine Konstante so hingebogen, dass sie genau die passenden Kräfte in Abhängigkeit von der Masse ergibt. Genau so geht Wissenschaft.

  141. #141 Kallewirsch
    6. Mai 2014

    Aber selbst die gesamte Rotverschiebung lässt sich viel plausibler und passender zu den gemessenen Werten erklären. z.B.:
    https://articles.adsabs.harvard.edu/full/1993ApJ…405…51N
    oder:
    https://redshift.vif.com/JournalFiles/V13NO2PDF/V13N2HAR.pdf

    Und das nennst du plausibel?

    The cosmological redshift is described by the intersection of
    two Hubble spheres, where a Hubble sphere is defined as a
    range over which spherical, quantum-waves interact,
    specifically Ru = 1.9 × 10^26 m.

    sphärische Quanten-Wellen. Ja klar. Quanten sind ja momentan groß in Mode.

    Ich hab nichts gegen andere Sichtweisen. Aber lass uns doch bei dem bleiben, was gesichert ist. So wie die Stringtheorie hat auch dieses Zeug genau 1 Problem. Niemand hat bisher diese Quantenwellen oder auch Hubble-Sphären nachgewiesen oder auch nur irgendeinen plausiblen Grund angeben können, warum es die geben soll.

  142. #142 Kallewirsch
    6. Mai 2014

    Dann wollen wir mal…

    Wenn ich mir zuerst die Website dieses Living-Universe angesehen hätte, dann hätte ich mir die Arbeit sparen können, diesen Sermon durchzulesen. Sehr empfehlenswert, sich zuerst mal den Rest durchzuarbeiten. So, so. Es gibt keine Gravitation. Statt dessen wird der Erdboden immer weiter nach ‘oben’ beschleunigt. Weil sich der Raum in der Erde ausdehnt (oder doch nicht? an anderer Stelle klingt es wieder eher so, als ob die Masse der Erde zunehmen würde – aber so genau hab ich dann auch nicht nachgelesen).
    Nur soviel am Rande: Wenn der Erdboden konstant mit 1g nach oben beschleunigt, dann erreicht er nach einem Jahr mehr oder weniger Lichtgeschwindigkeit. Und das hätte ganz dramatische Konsequenzen darüber, wie wir den Rest des Universums wahrnehmen.

    So, so. Ein Elektron war also das Anti-Teilchen zum Proton und die beiden gab es schon immer. Das kann aber nicht sein. Ein Elektron hat keine innere Struktur. Ein Proton schon, es besteht aus 3 Quarks. Ausserdem zerstrahlt Elektron und Proton nicht, wenn sie aufeinander treffen. Wäre dem so, dann hätte man bei den alten Röhrenfernsehern aber schön doof aus der Wäsche geguckt.

  143. #143 Alderamin
    6. Mai 2014

    @Kallewirsch

    Wenn ich mir zuerst die Website dieses Living-Universe angesehen hätte, dann hätte ich mir die Arbeit sparen können, diesen Sermon durchzulesen.

    Ach du lieber Himmel… das kann doch kein erwachsener Mensch ernst meinen.

    @max
    Wenn Du solche Leute als Deine Autoritäten und Referenzen anbringst, wirst Du niemanden hier überzeugen können.

  144. #144 Adent
    6. Mai 2014

    Und schwupps ist der Max wieder weg oder was?
    Ich hatte schon so ein ungutes Gefühl bei dem Begriff The Living Universe, nach dem Lesen desselben hat sich dieses Gefühl nicht verflüchtigt, ganz im Gegenteil, es erklärt warum der liebe Jim Carter lieber Bücher schreibt als sich mit dem Versuch eines Papers lächerlich zu machen.

  145. #145 max
    7. Mai 2014

    Zuallererst herzlichen Dank für die vielen Antworten!
    Wie man sieht werden in diesem Forum auch alternative Beiträge nicht ignoriert, das finde ich sehr dankenswert.

    Ich versuche auf alle Antworten in diesem einen Post einzugehen.

    Die Webseite: https://www.circlon.com/living-universe/054-The-Big-Bang-Paradoxes.html habe ich deshalb verlinkt, weil hier viele Paradoxien die allgemein mit der Urknall Theory diskutiert werden auf einer einzigen Seite aufgelistet sind. Das sind nicht nur die Einwürfe von James Carter…
    Es gibt ja auch noch vieles mehr zu berichten, aber dann müsste man viele Arbeiten und Links zusammenfuzzeln.
    Alderamin hat diesen Text als nicht sehr überzeugend befunden und vielem detailiert widersprochen. Mit meinem Erkenntnisstand kann ich davon einiges nicht zufriedenstellend bewerten. Daher werde ich wohl bei Gelegenheit die Kritik von Alderamin übersetzen und an James Carter schicken. Ich wäre nämlich auch gespannt, inwieweit darauf im einzelnen geantwortet werden kann, aber ich werde natürlich auch über weitere Quellen nachfragen.

    Ansonsten finde ich Carters Theory höchst amüsant zu lesen;) Ehrlich gesagt sind für mich aber Gravitonen auch nicht extrem viel besser. Wie schon gesagt informiere ich mich gerne in alle Richtungen und möchte auch diese Erfahrungen nicht missen. Wenn man sich für das Thema interessiert geht einem mit nur einer Theory schnell der Lesestoff aus. Egal ob richtig oder falsch, hier hat sich jemand viele Gedanken und viel Arbeit gemacht. Und es geht dabei auch nicht um Homöopathie, Schüsslersalze, Bachblüten oder andere Kassenschlager.

    Mit diesem extremen Beispiel möchte ich aber auch eine für mich sehr wichtige Aussage machen, mit dem Versuch damit einige Posts zu beantworten. Es gibt keinen Grund auf jemanden, der sich in der Sache bemüht hinzuhacken. Egal ob richtig oder falsch. Vielleicht inspiriert auch nur ein winziger Teil einer noch so abwegigen Theory dann doch denjenigen, der die Lösung findet. Jede heute anerkannte Theory war zuvor eine alternative Theory und wurde im besten Fall belächelt. So auch die Urknall Theory.

    In einem peer reviewed A-Journal möchte jeder Wissenschafter gerne publizieren. Aber nicht einmal B- und C-Journale nehmen Artikel auf, die nicht massenkonform oder/und gewinnversprechend sind. Von der in der Neuzeit stetig steigenden Freiheit der Wissenschaft ist heute leider nicht mehr viel zu sehen. Es ist wieder immer mehr Mode geworden, Andersdenkende zu beschimpfen statt einfach dankbar von ihren Arbeiten zu profitieren.

    Die Mainstream-Physik ist selbst absurd genug um sich über andere Theorien nicht lustig machen zu dürfen. Ein einziges Teilchen das einen Wellencharakter hat ist für mich damals dann genug gewesen um mich vom Glauben an den Mainstream abzukoppeln. Seither sehe ich mir alles an und eine sphärische, stehende Welle statt eines Teilchens erscheint mir heute viel plausibler, daher präferiere ich auch z.Zt. WSM. btw bei dieser Theory wäre ich für Falsifikationen dankbar.

    Von der Philosophie zurück zur Physik:)

    Ich poste nochmals die Links bezüglich der Rotverschiebung die ich gerne präsentieren wollte (ich hoffe diesmal klappts):

    Keine beschleunigte Expansion des Univerums:
    Tsagas
    https://www.livescience.com/33522-accelerating-universe-dark-energy-illusion.html

    Gar keine Expansion:
    Narlikar & Arp
    https://articles.adsabs.harvard.edu/full/1993ApJ…405…51N
    oder hoffentlich so:
    https://articles.adsabs.harvard.edu/cgi-bin/nph-iarticle_query?bibcode=1993ApJ…405…51N&db_key=AST&page_ind=5&plate_select=NO&data_type=GIF&type=SCREEN_GIF&classic=YES

    Harney
    https://redshift.vif.com/JournalFiles/V13NO2PDF/V13N2HAR.pdf

    Und für alle Mitleser poste ich gerne noch einmal diese einfach lesbare Seite mit Unterlinks um die Gegenbewegung zusammenzufassen, und mit Interesse an Kommentar:
    https://www.physikgrundlagen.de/universum.html

    Die Überschrift der Arbeit von Harney wurde von Kallewirsch besonders kritisiert aber diese Theory ist gerade mein persönlicher Favorit. Die sphärischen Quantenwellen wurden zwar noch nicht nachgewiesen, aber könnten wirklich extrem viel (im Verhältnis zum Standartmodell praktisch alles) erklären. Eine echte Theory für alles. Daher möchte ich die WSM Theory besonders hervorheben und hier noch einen Link posten:
    https://www-conf.slac.stanford.edu/einstein/talks/wolff.pdf

    Jedenfalls ist für mich der Urknall schon Geschichte:)

    Aber die Reise geht natürlich immer weiter, z.B. finde ich interessant:
    https://vixra.org/pdf/1302.0054v3.pdf

    P.S. habe mich gewundert, dass anscheinend niemand James Carter kannte. Streunt ihr wirklich nie herum? Keine Lust/Zeit? Dann versäumt ihr aber einiges;)

  146. #146 max
    7. Mai 2014

    der Link für Narlikar & Arp funktioniert anscheinend wieder nicht. Bitte einfach in Google folgenden Suchstring eingeben:
    1993ApJ…405…51N
    und den ersten vorgeschlagenen Link wählen. Unter “Free Fulltext Article” kommt man dann zum gesamten Text.

  147. #147 Florian Freistetter
    7. Mai 2014

    @max: Hättest du auch aktuelle Forschungsartikel die deine Thesen belegen oder nur ein paper aus dem Jahr 1993? Dir ist schon klar, was sich in den letzten 20 Jahren in der Kosmologie getan hat (bzw. vermutlich ist es dir nicht klar, sonst würdest du ja nicht deiner Verschwörungstheorie anhängen).

  148. #148 Alderamin
    7. Mai 2014

    @max

    Mit meinem Erkenntnisstand kann ich davon einiges nicht zufriedenstellend bewerten.

    Das ist Dein Problem. Und das Problem von James Carter. Um die Korrektheit der aktuellen Theorien bewerten zu können, reicht es nicht, in ein paar populärwissenschaftlichen Büchern darüber gelesen zu haben. Die Leute, die sich mit so was beschäftigen, haben Jahrzehnte gelernt und leiten ihre Theorien mit dem erforderlich mathematischen Handwerkszeug ab, das Du nicht hast, Herr Carter nicht, und die meisten hiet, mich eingeschlossen, ebenfalls nicht. Es steht uns nicht zu, da mitreden zu wollen, weil wir keine Ahnung haben. Die Leute, die eingereichte Papers bewerten, haben diese Ahnung, die sind vom Fach. Die können auch beurteilen, ob jemand Unsinn schreibt oder nicht. Es hat nichts mit “Unterdrückung anderer Meinungen” zu tun, wenn offensichtlicher Unsinn von Fachleuten abgewiesen wird. Was man keinesfalls abweisen kann, sind mathematisch nachvollziehbare Ableitungen und Messungen. Mr. Carter hat derlei aber nicht anzubieten.

    Z.B. wie lange welche Temperatur und Dichte im primordialen Gas geherrscht hat, um dann eine bestimmte Mischung von Elementen herauszubekommen. Man hat dann lediglich zwei freie Parameter, die Expansionsgeschwindigkeit und die Menge an dunkler Materie.

    Und das kommt dabei heraus. Man erhält eine Graphik, wo die relativen Häufigkeiten der Elemente an genau einer Stelle über 9,5 Größenordnungen genau mit der Messung der Menge der dunklen Materie durch WMAP (an der Hintergrundstrahlung) übereinstimmt. Fusionprozesse sind sehr gut verstanden. Diese Kurve beweist, dass es dunkle Materie gibt und dass die Elemente nicht in einem Stern gebildet wurden, sondern während einer kurzzeitigen Fusionsphase in einem heißen Feuerball. Das ganze dauerte nur 2 Minuten, wie diese Graphik zeigt.

    Es dürfte unmöglich sein, diese Ergebnisse durch andere Prozesse zu erklären als durch die Kernfusion in einem expandierenden Feuerball mit einem bestimmten Anteil dunkler Materie. Das ist einer der Beweise, dass der Urknall tatsächlich stattgefunden hat. Die ganzen Kritiker können diese Kurve nicht reproduzieren. Wenn sie das hinbekommen, kann man mal mit ihnen reden.

  149. #149 Adent
    7. Mai 2014

    @Max

    In einem peer reviewed A-Journal möchte jeder Wissenschafter gerne publizieren. Aber nicht einmal B- und C-Journale nehmen Artikel auf, die nicht massenkonform oder/und gewinnversprechend sind.

    Sie arbeiten/publizieren in welcher Wissenschaft? So wie sie das dargestellt haben ist es eine schlichtweg falsche Behauptung ohne Beleg.

    Ein einziges Teilchen das einen Wellencharakter hat ist für mich damals dann genug gewesen um mich vom Glauben an den Mainstream abzukoppeln.

    Also seit dem Doppelspaltexperiment oder was heißt damals?

  150. #150 Alderamin
    7. Mai 2014

    @max

    Ein anderer Beweis ist die Feinstruktur der Hintergrundstrahlung. Aus dem Urknall folgt, bei der richtigen Wahl der Parameter (in Übereinstimmung mit der Elementhäufigkeit!), eine gewisse Abhängigkeit von Temperaturschwankungen in gewissen Winkelabständen. Z.B. kann sich nach dem Urknall eine Inhomogenität (eine zufällig etwas wärmere Stelle) nur mit Lichtgeschwindigkeit durch Strahlung ausgleichen. Aus dem Alter der Hintergrundstrahlung von 380000 Jahren folgt, dass sich Inhomogenitäten nur über 380000 Lichtjahre ausgeglichen haben können, also Homogenität nur innerhalb maximal solcher Radien zu erwarten ist. In der Hintergrundstrahlung entspricht dies einem Sehwinkel von 1°. Man erwartet im Abstand von ca. 1° die letzte hohe Korrelation; Winkel darüber sollten eine geringe Korrelation haben (statistisch unabhängig sein).

    Die von PLANCK und vorher WMAP gemessene Korrelationskurve sieht so aus (1. Graphik). Die roten Punkte sind die Messung von PLANCK und die durchgezogene Linie ist die Vorhersage aus der Urknalltheorie (in die Peaks bei kleineren Winkeln gehen weitere Effekte ein, siehe hier). Die Übereinstimmung von Vorhersage und Messung ließ den Kollegen der Autorin des Artikels Freudensprünge vollführen.

    Das ist die Stärke der Wissenschaft. Die überlegt sich nicht nur nette Ideen und denkt, hmm, klingt gut, könnte stimmen, sondern sie trifft Annahmen, macht Voraussagen und weist diese dann durch Messungen nach. Genau das tun die ganzen Kritiker aber nicht.

    Frag’ Mr. Carter mal, wie er diese Ergebnisse reproduziert. Oder lieber nicht, wenn ich an den Blödsinn mit der beschleunigten Erde denke…

  151. #151 Alderamin
    7. Mai 2014

    @max

    Noch zu Deinen Links:

    Ich poste nochmals die Links bezüglich der Rotverschiebung die ich gerne präsentieren wollte (ich hoffe diesmal klappts):

    Keine beschleunigte Expansion des Univerums:
    Tsagas

    Er sucht das gemessene Rotverschiebungsprofil durch eine Bewegung unserer Raumregion durch den Weltraum zu erklären. Ich habe nicht verstanden, wie das funktionieren soll, warum man das an der Hintergrundstrahlung nicht sieht, und wie er dann das Alter der ältesten Sterne erklären will (ohne Dunkle Energie ist das Weltall jünger), aber er zweifelt am Urknall an sich gar nicht. Da wir nicht wissen, was Dunkle Energie ist, ist es legitim, nach Mechanismen jenseits einer Vakuumenergie zu suchen.

    Gar keine Expansion:
    Narlikar & Arp

    “aged light” erklärt nicht, warum Supernova-Kurven Zeitdilatation zeigen. Siehe hier Links zu entsprechenden Papieren.

    Harney

    “Hubble Spheres”: gleiches Argument, wenn die Rotverschiebung nicht aus Expansion herrührt, dann gäbe es keine nachweisbare Zeitdilatation.

    Die Überschrift der Arbeit von Harney wurde von Kallewirsch besonders kritisiert aber diese Theory ist gerade mein persönlicher Favorit. Die sphärischen Quantenwellen wurden zwar noch nicht nachgewiesen, aber könnten wirklich extrem viel (im Verhältnis zum Standartmodell praktisch alles) erklären. Eine echte Theory für alles.

    Auch für die Elementhäufigkeit und das Leistungsspektrum der Hintergrundstrahlung? Dunkle Materie und Energie wird von Dir abgelehnt, aber kosmologische Quantenwellen werden geschluckt. Warum?

    Und für alle Mitleser poste ich gerne noch einmal diese einfach lesbare Seite mit Unterlinks um die Gegenbewegung zusammenzufassen, und mit Interesse an Kommentar:

    Da ist der erste Satz schon falsch:

    Dies ist ein bewusster Gegenentwurf zur gegenwärtig in der Kosmologie vorherrschenden Urknall-Hypothese, die auf Modellvorstellungen aus den Zwanzigerjahren des letzten Jahrhunderts basiert, und die viele neuere Beobachtungen nicht physikalisch erklären kann.

    Die Beobachtung der Galaxienflucht erfolgte in den späten 20ern, aber an der Urknalltheorie hat man seitdem stetig weiter gearbeitet: fortwährende Messungen der Rotverschiebung mit zunehmender Genauigkeit, die Vorhersage der Hintergrundstrahlung 1958 und ihre Entdeckung 1964, die Inflation ca. 1980, die Dunkle Materie in den 80ern und die Dunkle Energie in den 90ern, die Messungen der Feinstruktur der Hintergrundstrahlung in den letzten 10 Jahren. Wie an den oben verlinkten Diagrammen zu sehen ist, erklärt nur die Urknalltheorie die allerneuesten Messungen.

    Zu dem weiteren Text könnte ich noch eine Menge schreiben, aber ich hab’ heute schon zu viel Zeit damit verplempert, vielleicht heute abend oder ein andermal. Aber wozu eigentlich, wird Dich ja eh’ nicht überzeugen. Kognitive Dissonanz.

    Aber die Reise geht natürlich immer weiter, z.B. finde ich interessant

    Reine Zahlenspielerei. Wenn man lange genug herumspielt, findet man alle möglichen scheinbaren Zusammenhänge. Ohne Theorie dahinter ist das nichts wert.

    Daher möchte ich die WSM Theory besonders hervorheben und hier noch einen Link posten:

    Lass’ uns doch bitte beim Thema Urknalltheorie bleiben.

  152. #152 Kallewirsch
    7. Mai 2014

    Von der in der Neuzeit stetig steigenden Freiheit der Wissenschaft ist heute leider nicht mehr viel zu sehen. Es ist wieder immer mehr Mode geworden, Andersdenkende zu beschimpfen statt einfach dankbar von ihren Arbeiten zu profitieren.

    Ich sehe das so, wie die Erforschung der Welt durch die Seefahrer im 16., 17. und 18. Jahhundert.
    Damals wusste man noch nicht viel darüber, wo welche Inseln sind, ob es noch Kontinente gäbe, die man nicht kennt. Die Landkarten enthielten noch viele weiße Flecken. Man konnte spekulieren und Hypothesen aufstellen, so viel man wollte.
    Heute ist das anders. Die Landkarte ist mehr oder weniger komplett gefüllt (es mag noch ein paar weiße Flecken irgendwo geben, meinetwegen im Amazonas-Dschungel oder in den Wüsten, an denen tatsächlich noch nie ein Mensch war), aber wenn mir heute jemand erzählt, es gäbe einen bisher noch unentdeckten Kontinent südlich des 70° südlichen Breiten-Grades, auf dem Menschenfresser hausen, dann brauch ich nicht selber hinfahren um zu wissen, dass er Unsinn plappert. Wer so etwas behauptet, darf sich nicht wundern, wenn er Gegenwind erhält.
    Und genau so ist es auch in der Physik. Die ‘Landkarte’ der Physik ist zwar bei weitem nicht vollständig. Aber gute Teile dieser Karte sind soweit abgesichert, dass man heutzutage mit Sicherheit sagen kann: das was der da plappert kann noch so gegen den Mainstream sein, es so einfach nicht richtig. Die Belege sind zu gut, als das wir annehmen müssten, das die bisherige Einfärbung der Karte an dieser Stelle falsch wäre.

  153. #153 A_Steroid
    7. Mai 2014

    @Kalle und Max
    abgesehen von dem “like” für Kalle noch in dem Zusammenhang der Hinweis: wenn jemand tatsächlich der Meinung ist, das südlich des 70° Breitengrades ein unentdeckter Kontinent lauert – kann er das an tausend Stellen veröffentlichen. ABER! Er sollte schon Fotos, GPS-Koordinaten usw. angeben. Dann werden dutzende von Schiffen / Flugzeugen das Ziel aufsuchen und die Mainstream-Forscher werde das begeistert feiern. So wie sie übrigens jede neu entdeckte Art feiern (aktuell in der Schweiz !!!!! eine neue Wespenart – die bisher niemand beachtet hat). Also unterdrücken würde den niemanden… Leider hapert es bei den “Querdenkern” häufig an der Beweislage… siehe Homöopathie o.ä. – denn behaoten kann ich vieles…

  154. #154 Alderamin
    8. Mai 2014

    @max

    Wenn Du noch mitliest: Das gestern abend Veröffentlichte passt doch hervorragend zu dem, was ich gestern morgen geschrieben habe:

    https://www.spiegel.de/wissenschaft/weltall/computer-simulation-illustris-zeigt-geschichte-des-universums-a-968135.html

    https://skyweek.wordpress.com/2014/05/08/die-ehrgeizigste-simulation-des-universums/

    Mit dem bekannten Mix aus Gas und dunkler Materie gelang es, unter Berücksichtigung der bekannten Naturgesetze, die Entstehung der Struktur des heutigen Universums inklusiver aller bekannten Galaxientypen zu simulieren. Ich würde mal sagen, dem kann kein Kritiker an der DM oder dem Urknall noch irgendetwas entgegensetzen. Case closed (nicht, dass er vorgestern noch offen gewesen wäre).

  155. #155 Alderamin
    8. Mai 2014
  156. […] Urknall – konnte diese Phase 2014 durch Beobachtungen nachgewiesen werden. Ich habe das alles hier und hier schon ausführlich zusammengefasst und möchte die entsprechenden Stellen aus Ferreiras […]

  157. #157 max
    13. Juni 2014

    Was für den einen Beweise sind, sind für den anderen Fehlinterpretationen:

    https://science.orf.at/stories/1739597/

  158. #158 Alderamin
    13. Juni 2014

    @max

    Wissen wir schon lange (“Eine ernsthafte Alternativ-Deutung für die BICEP2-Daten”). Ein angezweifeltes Ergebnis beim Nachweis der Inflation heißt aber weder, dass sie nicht stattfand, noch, dass es keinen Urknall gab oder keine DM gibt. Es heißt nur, dass die Veröffentlichung des Nachweises voreilig war (die Autoren sind aber noch überzeugt, dass ihre Ergebnisse stimmen). Noch in diesem Jahr werden die Daten von PLANCK erwartet, die sind genauer, dann wissen wir mehr.

    Dieses als Beispiel für die Fehlbarkeit der Wissenschaft hoch halten zu wollen, zieht nicht, denn es zeigt gleichzeitig, wie sie sich selbst korrigiert, und das ist gerade die wissenschaftliche Methode.

  159. #159 max
    6. Juli 2014

    Hallo!
    Ich melde mich mal wieder mit weiterem Material:
    https://www.youtube.com/watch?v=hC_KkLvG22A

    Stephen Crothers hat ja auch kürzlich für Stephen Hawking aufgelegt:
    https://www.principia-scientific.org/the-end-of-big-bang-black-holes-and-the-science-of-stephen-hawking.html

    Und Hawking hat auch gleich ünernommen:
    https://www.dailymail.co.uk/sciencetech/article-2545552/Stephen-Hawking-admits-no-black-holes-GREY-holes.html#ixzz2rMuEJlbh

    Gibt es von Eurer Seite etwas Neues?

  160. #160 max
    10. Februar 2015

    Ich weiß nicht ob euch das Thema überhaupt noch interessiert, aber von mir mal wieder eine Botschaft vom Urknall 😉
    https://phys.org/news/2015-02-big-quantum-equation-universe.html

    lg
    max

  161. #161 Galaxy
    Karlsruhe
    22. März 2015

    Hallo zusammen,
    Ich habe da mal eine grundsätzliche Frage zum Horizontproblem. Ich mag das ganze vielleicht etwas zu naiv sehen, aber ich verstehe nicht richtig, warum damit ein Problem verbunden ist. Es wird immer wieder auf folgendes hingewiesen, wie auch hier bei Florian:
    “…Dabei geht es um die Frage, warum das Universum so enorm homogen ist. Egal in welche Richtung man schaut, es sieht überall gleich aus… Das ist nur dann möglich, wenn sich auch die ursprüngliche Materie, die kurz nach dem Urknall das Universum erfüllt hat, entsprechend homogen verteilt war und das wiederum ist nur möglich, wenn diese Materie ausreichend Zeit hatte, sich genügend zu vermischen. Das junge Universum war zwar viel kleiner als heute, aber wenn man von einer normalen Expansionsrate ausgeht, dann war es trotzdem zu keinem Zeitpunkt klein genug als das das eine “Ende” des Universums wissen konnte, was im anderen “Ende” passiert.”

    Das junge Universum war ja eben infinitesimal klein, also ursprünglich sowieso homogen. Ich sehe nicht, warum es zu keinem Zeitpunkt klein genug hätte sein sollen, damit das eine “Ende” des Universums nicht hätte wissen können, was im anderen “Ende” passiert. Habe ich eine homogene Menge Masse oder Energie, dann kann sie sich doch bei einer Explosion (Urknall) eben auch homogen und isotrop verteilen. Dementsprechend weiß das eine “Ende” nach meiner Theorie sehr wohl, was im anderen “Ende” passiert ist, nämlich das gleiche, weil sie eben aus der homogenen Menge der “Ursprungssuppe” entstanden sind.

    Würde Euren Kommentar wirklich schätzen – Danke.

  162. #162 Alderamin
    23. März 2015

    @Galaxy

    Zunächst: Es gibt in der Kosmologie einen Parameter “Skalenfaktor“, der angibt, um welchen Faktor eine Strecke im Universum zu anderen Zeiten als heute aufgrund der kosmischen Expansion gestreckt oder gestaucht war. Für eine Rotverschiebung z beträgt der Skalenfaktor a=1/(1+z). Die kosmische Hintergrundstrahlung hat etwa eine Rotverschiebung von 1100 (2,7 K statt der ursprünglichen 3000 K des heißen Plasmas an der Schwelle zur Rekombination), d.h. der Skalenfaktor betrug damals, 380000 Jahre nach dem Urknall, 1/1100 gegenüber heute. Damit durchmaß das beobachtbare Universum (heute: 90 Milliarden Lichtjahre Durchmesser) nur rund 82 Millionen Lichtjahre, aber eben weit mehr, als 380000 Lichtjahre, die ein Lichtstrahl seit dem Urknall hätte zurücklegen können. Deswegen konnte kein Temperaturausgleich zwischen hinreichend weit getrennten Regionen über den Austausch von Strahlung stattfinden.

    Vom Start aus völlig identisch können die Temperaturen aber auch nicht gewesen sein, denn auf engstem Raum verbieten die Gesetze der Quantenmechanik, dass die Energie über kurze Zeiten überall gleich groß war, es muss also vom Start aus Schwankungen gegeben haben (selbst wenn alles in einem Punkt begonnen hätte, wäre daraus innerhalb kurzer Zeit ein kleines Volumen mit solchen Schwankungen entstanden, die dann nicht mehr ausgeglichen werden konnten).

    Bei der Inflation geht man davon aus, dass vorher noch ein Ausgleich stattfinden konnte und die Expansion dann ruckartig das Unviersum stark vergrößerte, so dass die ausgeglichenen Regionen weit voneinander getrennt wurden. So wird das Horizontproblem gelöst.

  163. #163 Galaxy
    Karlsruhe
    23. März 2015

    Danke Alderamin für die Mühe und die ausführlichen Erläuterungen.
    Ich darf vielleicht mit zusätzlichen Anregungen um weitere Diskussion bitten. Ich nummeriere einmal der Übersichtlichkeit wegen:

    1) Deiner logischen Darlegung unter Berücksichtigung des Skalenfaktors kann ich folgen, nicht zuletzt durch Rückerinnerung und weiterführendes Studium zum Thema. Jedoch – ist denn die Gesamtthematik um den Skalenfaktor hinreichend als quasi unumstößlich bewiesen? Möglicherweise wären “Anpassungen” zu überlegen, die aus bislang nicht genügend untersuchten Verschiebungen in den dazugehörigen Parametern, die zu den Berechnungen führen, herrühren. Was ich konkret meine, vielleicht stimmt an der durchgängigen Überlegung um den Skalenfaktor irgendwo etwas nicht genau. Wir glauben ja heute auch beispielsweise zu wissen, dass das Universum sich in den Epochen mit unterschiedlichen Geschwindigkeiten ausdehnte.

    2) Deinen zweiten Absatz kann ich nachvollziehen und hier zunächst einmal als gegeben übernehmen. Aber warum kann man denn bei der Inflation einfach davon ausgehen, dass “vorher noch ein Ausgleich stattfinden konnte” – nur weil wir heute ein mehr oder weniger homogenes Weltall vorfinden? Das ist ja eigentlich aus der Wirkung lediglich die Ursache vermutet. Oder ist die Inhomogenität lediglich ein Problem der Quantenmechanik für Gegebenheiten auf engstem Raum, während bei Zuständen in zeitlich “postquantenmechanischen” Räumen (bezogen auf den Urknall) eine Vermischung jederzeit möglich ist?

    3) Die Inflationstheorie ist zwar sicherlich ein recht genialer Ansatz zur Lösung des Horizontproblems, aber warum um alles in der Welt soll sich denn das Universum in der angedachten Phase ruckartig vergrößert haben, außer zur Erklärung des Horizontproblems? Gibt es hierzu denn irgendwelche beweisähnlichen Beobachtungen/Messungen etc.? Hört sich das denn nicht irgendwie ein wenig nach Ptolemäischen Erklärungsversuchen an? – die waren ja zunächst auch gar nicht so schlecht 😉

    4) Akzeptieren wir aber hier einmal die Inflationstheorie, dann muss ich doch davon ausgehen, dass diese ruckartige Expansion mit Geschwindigkeiten >c erfolgte. Wie verträgt sich denn das mit der ART?

  164. #164 Nebukadnezar
    23. März 2015

    Der Raum kann sich mit jeder beliebigen Geschwindigkeit lokal ausdehnen. ART ist über lokale Ereignisse in der Raumzeit, und nicht über den Raum an sich.

  165. #165 Orci
    23. März 2015

    @ Galaxy:

    Zu 3: Wenn eine Flüssigkeit verdampft, entzieht sie ihrer Umgebung Wärme (deswegen wird Dir Kalt, wenn Du aus dem Wasser steigst, ganz egal wie heiß die Sonne brennt). Wenn ein Gas kondensiert, gibt es Wärme an seine Umgebung ab – deswegen ist der Radiator hinten am Kühlschrank heiß.
    Die Inflation kann man für Fußgänger als solchen Phasenübergang beschreiben: An einem bestimmten Punkt wurde schlagartig sehr viel Energie frei, die das Universum außeinander getrieben hat.

    Die Experten mögen mir für dieses vereinfachte Bild vergeben.

  166. #166 Alderamin
    23. März 2015

    @Galaxy

    Jedoch – ist denn die Gesamtthematik um den Skalenfaktor hinreichend als quasi unumstößlich bewiesen?

    Das ist ein elementarer Bestandteil der Urknalltheorie. Belegt (beweisen geht in der Naturwissenschaft nicht, nur ind er Mathematik) ist jedenfalls

    – dass der Raum expandiert (Supernovae-Entfernungsmessungen; der Hubble Paramter ist gerade die Ableitung des Skalenfaktors nach der Zeit dividiert durch den Skalenfaktor)
    – dass Galaxien bei großer Rotverschiebung wie mit der Lupe vergrößert erscheinen
    – dass Abhängigkeiten in der Hintergrundstrahlung genau bei dem Winkelabstand ihre Grenze finden, der der Lichtlaufstrecke entspricht, die dem Alter der Hintergrundstrahlung entspricht (genau das ist nämlich der große Peak bei 1° im Spektrum der Hintergrundstrahlung, die die Intensität von Korrelationen über dem Winkelabstand aufträgt; wobei die 1° schon den “Lupenfaktor” beinhalten, denn 380000 Lichtjahre haben auch schon in der lächerlichen Entfernung von 21,8 Millionen Lichtjahren entsprechend diesen Winkeldurchmesser – bei einer Rotverschiebung von z=1100 dann wieder)

    Aber warum kann man denn bei der Inflation einfach davon ausgehen, dass “vorher noch ein Ausgleich stattfinden konnte” – nur weil wir heute ein mehr oder weniger homogenes Weltall vorfinden? Das ist ja eigentlich aus der Wirkung lediglich die Ursache vermutet.

    Ausgehend von den Fragen “warum ist die Temperatur ausgeglichen, wo sie es klassisch nicht sein dürfte?”, “warum ist der Raum heute noch im Rahmen der Messgenauigkeit flach, obwohl jede kleinste Abweichung von der Flachheit über die Milliarden Jahre extrem zugenommen haben müsste?” und “warum findet man keine magnetischen Monopole?” fragten sich Alan Guth und Andrei Linde, wie sie dies alles unter einen Hut bringen konnten und zwar mit einer Theorie, die (wie alle Theorien) auf Bekanntem aufbauend (in diesem Fall die Allgemeine Relativitätstheorie) die Beobachtungen erklären konnten. Die ART erlaubt es dem Raum, unter Gravitation zu expandieren (und zwar sehr schnell), wenn im Vakuum ein gewisser Druck besteht (Druck geht mit in den Energie-Impuls-Tensor ein, über den Martin Bäker bei “Hier wohnen Drachen” aktuell sinniert).

    Gegeben ein hinreichend hoher (negativer) Druck im Vakuum, dann würde die ART eine Inflation erklären. Der neue und hypothetische Teil der Inflationstheorie ist der Ursprung dieses Drucks, und wie die Inflation dann wieder zum Stillstand kam (und genau darin unterscheiden sich verschiedene Varianten der Inflationstheorie, davon gibt es nämlich mehrere). Guth hat berechnet, dass es reicht, ein paar Kilogramm Materie auf ein Planckvolumen zu verdichten, um einen solchen Vakuumzustand zu erzeugen (in der Größenordnung von 10^60 g/cm³, wenn ich mich recht entsinne). Das ist nicht gerade wenig. Vielleicht leisten Schwarze Löcher so etwas, vielleicht eine Quantenfluktuation, wenn sie eine Unendlichkeit Zeit hat, zu entstehen, darüber kann man nur spekulieren. Aber die Inflationstheorie nennt einen Mechanismus, der die drei Eingangsprobleme (Horizont-, Flachheits- und Monopolproblem) alle lösen kann, und er ist im Rahmen der ART möglich. Außerdem erklärt er die Struktur der Hintergrundstrahlung (die wiederum die Quelle der großräumigen Struktur des heutigen Universums ist) als vergrößerte Quantenfluktuationen in der Energiedichte vor der Inflation, und macht überprüfbare Vorhersagen über die Polarisation der Hintergrundstrahlung. Eine bessere Alternative gibt’s derzeit nicht.

    In Kombination mit einer vor der Inflation bis auf Quantenfluktuationen ausgeglichenen Temperatur (und aus der Größe der Strukturen in der Hintergrundstrahlung kann man vermutlich auch darauf schließen, wann und bei welcher Größenordnung die Inflation dann loslegte – es ist die Rede von 10^-35 s nach der hypothetischen Singularität) kann man die heutigen Beobachtungen erklären.

    So geht die Naturwissenschaft aber eigentlich immer vor: aus den Beobachtungen auf einen möglichen Mechanismus schließen, der sie hervorbringen kann, und dann nach Seiteneffekten des Mechanismus suchen, die man belegen oder widerlegen kann.

    Also tl;dr: ja genau, weil wir heute ein homogenes Weltall vorfinden, ist die Inflation mit vorherigem Temperaturausgleich die bevorzugte Erklärung dafür.

    Die Inflationstheorie ist zwar sicherlich ein recht genialer Ansatz zur Lösung des Horizontproblems, aber warum um alles in der Welt soll sich denn das Universum in der angedachten Phase ruckartig vergrößert haben, außer zur Erklärung des Horizontproblems?

    Ist das mit obigen Errklärungen beantwortet? Also Lösung gleich dreier Probleme, Erklärung der Struktur der Hintergrundstrahlung, physikalische Plausibilität im Rahmen der ART. Hinzu kommt noch die artverwandte Dunkle Energie, die allerdings zu Zeiten von Guth und Linde noch unbekannt war. Sie ist eine Inflation in Superzeitlupe.

    Akzeptieren wir aber hier einmal die Inflationstheorie, dann muss ich doch davon ausgehen, dass diese ruckartige Expansion mit Geschwindigkeiten >c erfolgte. Wie verträgt sich denn das mit der ART?

    Na ja, die Theorie folgt ja gerade aus der ART. Die Frage ist eher, wie sie sich mit der SRT verträgt, die ja sagt, dass sich nichts schneller als mit Lichtgeschwindigkeit im Raum ausbreiten kann. Nun hat sich aber nichts durch den Raum bewegt, sondern der Raum ist einfach gewachsen, so wie er das heute auch tut. Galaxien stehen im Raum an ihren Orten fast still, aber der der Raum zwischen ihnen quillt auf, d.h. es bewegt sich nichts durch den Raum.

    Heute quillt er um einen Faktor 1 + 2,6*10^-18 pro Sekunde auf, aber auch damit kommt man über eine Strecke von 13,8 Milliarden Lichtjahren auf eine Zuwachsrate von 300000 km/s, und dies ist ja das, was wir tatsächlich live beobachten. Bei der Inflation wuchs der Raum alle 10^-35 s um den Faktor 2, also 2^2 nach 2*10^-35 s, 2^10 nach 10^-34 s und 2^100 nach 10^-33 s. Da soll die Inflation dann geendet haben, 2^100 sind etwa ein Wachstum um den Faktor 10^30, ich nehme an, das ist der Faktor, um den man Quantenfluktuationen vergrößern muss, damit sie danach bei normalem Hubble-Wachstum (mit dem damals deutlich höheren Hubble-Parameter, aber den kann man rückwärts ausrechnen) auf die Größe der Strukturen in der Hintergrundstrahlung kommt.

    Die Details, aus denen man die Dauer und Intensität dieses inflationären Rucks abgeleitet hat, weiß ich allerdings auch nicht, aber die Zahlen sind mit Sicherheit begründet.

    Ja, die Inflationstheorie verträgt sich hervorragend mit der Relativitätstheorie (der einen wie der anderen). Sie ist sogar in einer der beiden begründet.

  167. #167 Galaxy
    23. März 2015

    Uups – stimmt Nebukadnezar, danke der Erinnerung. Die Lorentztransformation gilt ja nur innerhalb eines Inertialsystems im Minkowsky-Raum.

  168. #168 Alderamin
    23. März 2015

    @Orci

    Die Inflation kann man für Fußgänger als solchen Phasenübergang beschreiben: An einem bestimmten Punkt wurde schlagartig sehr viel Energie frei, die das Universum außeinander getrieben hat.

    Eigentlich ist die Aussage gerade umgekehrt: als die Inflation endete, tunnelte das Vakuum auf eine niedrigere Energiestufe (Phasenübergang) und gab die Vakuumenergie als Strahlung frei. Dies bezeichnen die Kosmologen daher auch als Reheating.

  169. #169 Krypto
    23. März 2015

    @Alderamin:

    aber auch damit kommt man über eine Strecke von 13,8 Milliarden Lichtjahren auf eine Zuwachsrate von 300000 km/s

    Das ist mir noch gar nicht aufgefallen; wäre dies nicht ein unglaublicher Zufall…?

  170. #170 Alderamin
    23. März 2015

    @Krypto

    Kein Zufall, denn rückwärts interpoliert würden Objekte, die sich mit Lichtgeschwindigkeit entfernen und in 13,8 Milliarden Lichtjahren Entfernung befinden, ja vor 13,8 Milliarden Jahren genau mit uns einen gemeinsamen Ursprung teilen. Man nennt dies den Hubble-Radius (oder -Volumen). 13,8 Milliarden Lichtjahre ist aber nicht ganz richtig, weil die Expansionsrate nicht konstant war, es ist eigentlich etwas mehr, siehe verlinkten Artikel oder die Grafiken hier (es geht um die grüne Zone in den Diagrammen).

    Naiv gerechnet mit H0=71 km/s/Mpc komme ich jedenfalls auf 300000/71 * 3,26E+6 LJ = 13,775E+9 LJ.

  171. #171 Krypto
    23. März 2015

    Der Zufall, den ich sehe, ist das wir ausgerechnet zu der Zeit leben und beobachten sollen, zu der sich die Raumexpansion auf LG summiert…

  172. #172 Alderamin
    23. März 2015

    @Krypto

    Der Zufall, den ich sehe, ist das wir ausgerechnet zu der Zeit leben und beobachten sollen, zu der sich die Raumexpansion auf LG summiert…

    Das ist definitiv kein Zufall, sondern das tut und tat sie zu jeder Zeit für irgendeine Entfernung, und sie hört auch nicht bei c auf. Für 28 Milliarden Lichtjahre addiert sie sich auf 2c. Und als das Weltall halb so alt war, da war der Hubble-Radius auch in etwa halb so groß. Siehe die grünen Bereiche in den verlinkten Grafiken.

    Dass es jenseits der Hubble-Sphäre allerhöchstwahrscheinlich noch Raum gibt, liegt an der Inflation und/oder daran, dass das Universum möglicherweise nicht in einem Punkt begann, sondern von Beginn an unendlich groß war.

  173. #173 Niels
    23. März 2015

    @Alderamin

    Dass es jenseits der Hubble-Sphäre allerhöchstwahrscheinlich noch Raum gibt, liegt an der Inflation und/oder daran, dass das Universum möglicherweise nicht in einem Punkt begann, sondern von Beginn an unendlich groß war.

    Verwechselst du hier Hubble-Sphäre und Partikelhorizont?
    Natürlich ist das beobachtbare Universum größer als die Hubble-Sphäre. Das wäre es doch auch ohne Inflation und ohne Unendlichkeit des Gesamtuniversums.

    Kein Zufall, denn rückwärts interpoliert würden Objekte, die sich mit Lichtgeschwindigkeit entfernen und in 13,8 Milliarden Lichtjahren Entfernung befinden, ja vor 13,8 Milliarden Jahren genau mit uns einen gemeinsamen Ursprung teilen

    Dass c/H(t) in guter Näherung gleich c*t ist (H(t) ist der Hubble-Parameter, t ist das Universumsalter), ist aber doch für die meisten Zeiten t falsch.
    Dass diese Interpolation für den jetzigen Zeitpunkt ein brauchbares, ja sogar gutes Ergebnis liefert ist daher meiner Meinung nach durchaus Zufall.

  174. #174 Galaxy
    23. März 2015

    Danke an alle für manche Klärung meiner Fragen und Anregungen, vor allem auch für den langen Artikel von Alderamin.
    Jetzt bitte ich zusätzlich um folgende Klärung:

    Seit der Entdeckung der CBR wissen wir um die kleinen Variationen dieser Strahlung. Jetzt soll ja die Urknalltheorie genau diese Variationen vorhergesagt haben. Meine Fragen:

    1) Inwiefern hat die Urknalltheorie die Variationen in der CBR vorhergesagt?
    2) Diese Variationen sind ja eben der Beleg für eine Inhomogenität des Universums.
    Verstehe ich das so richtig, dass diese Inhomogenität insofern nichts mit der Frage der Homogenität des Universums zu tun hat, weil es sich beim CBR um ein Bild nach der Rekombination, eben 380000 Jahren ab Urknall handelt, während die Diskussion um die Homogenität des Universums sich auf den Zustand des Universums unmittelbar nach dem Urknall bezieht, will demnach heißen die Diskussion um die Homogenität des Universums nach dem Urknall hat nichts mit der Inhomogenität in der CBR zu tun?

  175. #175 Florian Freistetter
    23. März 2015

    @Galaxy: “Inwiefern hat die Urknalltheorie die Variationen in der CBR vorhergesagt?”

    Wenn du das wirklich ausführlich und verständlich nachvollziehen willst, empfehle ich dir das Buch “Big Bang” von Simon Singh. Im Prinzip geht es darum, dass es diese Strukturen im CMB geben MUSS, da ansonsten keine größere Strukturen wie Galaxien u,ä. entstehen können.

  176. #176 Alderamin
    23. März 2015

    @Niels

    Verwechselst du hier Hubble-Sphäre und Partikelhorizont?

    Es ging Krypto in #171 doch darum, dass sich die Expansionsgeschwindigkeit zufällig auf c addiert, und mein Punkt war, dass sie sich immer irgendwo auf c addiert, dass hinter dem Hubble-Radius nicht Schluss ist. Das gilt für den Partikelhorizont genauso (wo die Expansionsgeschwindigkeit allerdings nicht c ist).

    Dass c/H(t) in guter Näherung gleich c*t ist (H(t) ist der Hubble-Parameter, t ist das Universumsalter), ist aber doch für die meisten Zeiten t falsch.

    Na ja, exakt ist es heute ja auch nicht. Wenn man in dieser Grafik die Begrenzung des grünen Bereichs unterhalb der waagerechten schwarzen Linie durch eine Gerade von 0 bis zum Schnittpunkt des grünen Bereichs mit der schwarzen Linie ersetzt, dann erhält man doch eben jene Näherung, und die würde in der Vergangenheit im dargestellten Diagramm nicht weit von der grünen Begrenzungslinie zu liegen kommen; in der Zukunft wird’s allerdings ungenau (und das liegt, denke ich, an der Dunklen Energie, oder?). Kann sein, dass es näher am Urknall, als diese Grafik sichtbar macht, ebenfalls sehr ungenau war.

    Den Wiki-Artikel zur Hubble-Sphäre verstehe ich so (“This is because 1/H_0 gives the age of the universe by a backward extrapolation which assumes that the recession speed of each galaxy has been constant.”), dass das eben nicht zufällig der Fall ist. Zwar auch nicht exakt, das hängt natürlich davon ab, wie stark H(t) variiert hat, die Näherung wäre eben nur exakt für H(t) = const. = H0. Ich meine mich sogar zu erinnern, dass man in den späten 70ern / frühen 80ern immer so das Weltalter berechnet hatte. Da war die Dunkle Energie noch unbekannt und die Hubble-Konstante “50-100 km/s/Mpc”, da reichte die Genauigkeit völlig (10-20 Mrd. Jahre Weltalter)

    Natürlich ist das beobachtbare Universum größer als die Hubble-Sphäre. Das wäre es doch auch ohne Inflation und ohne Unendlichkeit des Gesamtuniversums.

    Es ging mir bei dieser Aussage um das Universum insgesamt, damit stets die Existenz eines Ortes gegeben ist, der mit 2c (oder auf Wunsch auch viel mehr) vom Beobachter fortstrebt. Nicht nur um das beobachtbare Universum, das weniger als dreimal größer ist als der Hubble-Radius. Zwischen Partikelhorizont und Hubble-Radius ist natürlich noch etwas Platz für ein Universum, das aus einem Punkt entstand und nie inflationär wuchs, da hast Du recht.

    Ich wollte es aber nicht zu kompliziert machen. Die Geschichte von H(t) und der Unterschied zwischen Lichtlaufzeitentfernung und Mitbewegter Entfernung frisst schnelle eine Bildschirmseite an Text.

  177. #177 Alderamin
    23. März 2015

    @Galaxy

    1) Inwiefern hat die Urknalltheorie die Variationen in der CBR vorhergesagt?
    2) Diese Variationen sind ja eben der Beleg für eine Inhomogenität des Universums.

    Die Temperatur der Hintergrundstrahlung ist überall 2,725 (± 0,002) K. Die Inhomogenitäten bewegen sich im Millionstel-Kelvin-Bereich.

    Einerseits sagte die Inflation voraus, dass unvermeidliche Quantenfluktuationen im Temperaturfeld auf winzigsten Distanzen (exakt gleich kann auch eine ausgeglichene Temperatur wegen der Heisenbergschen Unschärferelation auf mikroskopischen Abständen nie werden) durch die gigantische Expansion makroskopisch vergrößert worden sein sollten. Zum anderen gilt das, was Florian sagt, die großräumige Struktur des Universums verlangte Dichteschwankungen in der Hintergrundstrahlung als Ursprung.

    Man hat sich also von zwei Seiten getroffen: die Inflation sagte (schon vor COBE, WMAP und PLANCK) vorher, dass man in der Hintergrundstrahlung kleine Inhomogenitäten finden müsse, und die fand COBE in den 90ern zum ersten Mal. Und zum anderen sagte die Kosmologie voraus, dass es solche Unregelmäßigkeiten gegeben haben muss, damit sich Materie überhaupt lokal verdichten konnte, das passte prima zusammen.

    Heute läuft ein Projekt (las ich neulich in Sky & Telescope), bei dem man die Verteilung der Voids und Filamente großräumig untersuchen will, um aus diesen die gleichen Größen herauszulesen, die man mit WMAP und PLANCK aus dem Mikrowellenhintergrund herauslas. Eben die Werte jener Parameter, die das Lambda-CDM-Modell benötigt. Und diese Methode soll wohl noch genauere Ergebnisse liefern, hieß es. Die zugrundeliegende Struktur ist also dieselbe (nur jünger).

  178. #178 Jens
    23. März 2015

    Die Rotverschiebung der Hintergrundstrahlung beträgt z=1089.
    Wie kommt man darauf?

  179. #179 Niels
    23. März 2015

    @Alderamin

    und mein Punkt war, dass sie sich immer irgendwo auf c addiert, dass hinter dem Hubble-Radius nicht Schluss ist.

    Na ja, genau am Hubble-Radius addiert sie sich auf c. 😉
    Aber das weißt du natürlich.

    Das gilt für den Partikelhorizont genauso

    Allerdings nur mit Sicherheit, wenn es Inflation gab und/oder das Universum unendlich groß ist.
    Aber ich verstehe, was du sagen wolltest. Die Formulierung war allerdings meiner Meinung nach nicht besonders geglückt.
    Außerdem kennst du mich doch, ich bin ein unverbesserlicher Beckmesser.

    Na ja, exakt ist es heute ja auch nicht. […]
    in der Zukunft wird’s allerdings ungenau (und das liegt, denke ich, an der Dunklen Energie, oder?). Kann sein, dass es näher am Urknall, als diese Grafik sichtbar macht, ebenfalls sehr ungenau war

    Ja , liegt an der dunklen Energie. Und ja, nahe beim Urknall wird es ebenfalls sehr schlecht. Das liegt daran, dass die Expansion damals durch die Gravitation stark gebremst wurde.
    Zur Veranschaulichung habe [1/H(t)]/t mal geplottet, t in Milliarden Jahren nach dem Urknall:
    https://www.wolframalpha.com/input/?i=Plot%5B16.2441+Tanh%5B0.0923534+t%5D%2Ft%2C+%7Bt%2C+0%2C+50%7D%5D

    Für 0 bis ungefähr eine Million Jahre nach dem Urknall ist das aber völlig falsch, da taugt meine angenommene Näherung nichts mehr, weil ich die Strahlungsdichte vernachlässige.
    Da wird die korrekte Rechnung dann extrem kompliziert. Keine Ahnung, wie da der richtige Verlauf aussieht.

    Den Wiki-Artikel zur Hubble-Sphäre verstehe ich so (“This is because 1/H_0 gives the age of the universe by a backward extrapolation which assumes that the recession speed of each galaxy has been constant.”), dass das eben nicht zufällig der Fall ist.

    Na ja, wenn du dir das Schaubild ansiehst liegt y = 1 fast genau bei 14 Milliarden Jahren, also dem heutigen Universumsalter.
    Das liegt daran, dass die angesammelte Abbremsung und die neu aufsummierte Beschleunigung gerade im richtigen Verhältnis stehen. (Das ist jetzt ziemlich wischi-waschi, aber aufs Formeltippen habe ich gerade keine Lust.)

    Jetzt kann man sich natürlich darüber streiten, ob es wirklich Zufall ist, dass wir als bewusste Beobachter gerade zu diesem speziellen t = 14 das Universum untersuchen.

    Ich finde es aber, gerade wenn man den Plott betrachtet, sollte man als Nullhypothese erst einmal von einem Zufall ausgehen.
    Darauf wollte ich oben hauptsächlich hinaus, war aber mal wieder zu kurz formuliert.

    Das ist insgesamt aber eine Antwort auf:

    aber auch damit kommt man über eine Strecke von 13,8 Milliarden Lichtjahren auf eine Zuwachsrate von 300000 km/s

    Das ist mir noch gar nicht aufgefallen; wäre dies nicht ein unglaublicher Zufall…?

    Das ist wie gesagt meiner Ansicht nach Zufall.
    #171, in dem klar wird, dass Krypto etwas anderes gemeint hat, habe ich überlesen. Sorry.

  180. #180 Alderamin
    23. März 2015

    @Niels

    Ok, danke für die Klarstellungen.

    @Jens
    Zum einen kann man das aus der Temperatur ableiten, bei der das ursprüngliche Gas durchsichtig wurde. Vorher war es heiß und ionisiert, also ein Plasma (wie das Gas, aus dem die Sonne besteht) und Plasma ist undurchsichtig. Die letzte Strahlung verließ das Gas, als es gerade wieder zu durchsichtigem, neutralem Gas rekombinierte. Das war bei ca. 3000 K der Fall. Heute misst man 2,725 K und 3000/2,725 = 1100 (die Peak-Wellenlänge von Schwarzkörperstrahlung ist proportional zur Temperatur, Wiensches Verschiebungsgesetz).

    Kann aber sein, dass da noch mehr mit einfließt, die Funktion des Hubble-Parameters, die Dichte, die Menge an Helium, die während der Nukleosynthese entstanden ist, ich nehme an, das geht alles in eine Zustandsgleichung mit ein, die dann genau beschreibt, wann das Gas bis zur Rekombination abgekühlt war.

    Bei Galaxien kann man die Position der Wasserstofflinien messen, aber das geht bei der Hintergrundstrahlung, glaube ich, nicht.

  181. #181 Krypto
    24. März 2015

    @Niels, Alderamin:
    Danke für die vielen Klarstellungen. Genau so, wie Niels meine Frage zuerst aufgefasst hatte, habe ich es gemeint.
    Mir ist natürlich klar, dass sich die Expansion _unter anderem_ genau zu LG aufsummiert und auch darüber hinaus geht.
    Aber die Korrelation mit dem Alter des Universums finde ich durchaus zufällig.
    Was auch gar nicht weiter bemerkenswert sein muss, wenn nicht die “Gefahr” besteht, dass interessierte Laien diesen Zufall nicht erkennen und falsche Rückschlüsse auf entsprechende Modelle ziehen.
    Vielleicht liege ich als interessierter Laie jedoch falsch und Ihr sagt mir: “Das ist durchaus korrekt und alles andere als Zufall, dass sich die Expansion in 13,8Gly zu LG aufsummiert und das Alter des Universums 13,8Gy beträgt.”

  182. #182 Alderamin
    24. März 2015

    @Krypto, Niels

    Nach meiner Auffassung ist es nicht völliger Zufall, sondern es gibt ja einen Zusammenhang zwischen dem Hubble-Parameter und dem Weltalter. Wenn der Hubble-Parameter konstant wäre, müsste der Hubble-Radius der Lichtlaufzeit des Weltalters entsprechen. Nun ist der Hubble-Parameter nicht konstant, und insofern kommt es zufällig einigermaßen hin (14,2 Milliarden Lichtjahre Hubble-Radius, las ich gestern in einer der Quellen; 13,8 Milliarden Jahre Weltalter).

    Also, ich würde das nach wie vor nicht als “riesigen Zufall” bezeichnen sondern es gibt schon eine Korrelation. Aber es gibt eine zufällige Komponente, denn die integrierte Expansion macht aufgrund der Dunklen Energie eine Schlangenlinie um die Gerade herum, die einem konstanten Hubble-Parameter entspräche, so dass die Gerade die Entwicklung für eine lange Zeit gut approximiert.

    So würde ich meinen Standpunkt zusammenfassen.

  183. #183 Alderamin
    24. März 2015

    @Krypto, Niels

    es gibt ja einen Zusammenhang zwischen dem Hubble-Parameter und dem Weltalter

    Vielleicht macht ein Gegenbeispiel klarer, was ich damit meine: Es gibt keinen Zusammenhang zwischen der Größe und Entfernung des Mondes bzw. der Sonne. Trotzdem erscheinen beide im Rahmen der Schwankungen durch die ellipsenförmigen Orbits von Erde und Mond am Himmel fast genau gleich groß. Das ist ein riesiger Zufall nach meinem Verständnis. Weil eben gar kein Zusammenhang besteht.

  184. #184 Niels
    24. März 2015

    @Alderamin

    Nach meiner Auffassung ist es nicht völliger Zufall, sondern es gibt ja einen Zusammenhang zwischen dem Hubble-Parameter und dem Weltalter.

    Ja, natürlich. Nämlich H(t) = a’(t)/a(t). Das kann man dann nach t umstellen.

    Universumsalter t in Abhängigkeit vom Hubble-Parameter H für H von 60 bis 75
    Universumsalter t in Abhängigkeit vom Hubble-Parameter H für H von 75 bis 500
    Das ist aber kein linearer Zusammenhang, sondern eine Areakotangens-Hyperbolicus-Funktion.

    Wenn der Hubble-Parameter konstant wäre, müsste der Hubble-Radius der Lichtlaufzeit des Weltalters entsprechen.

    Wieso das denn?
    Der Hubble-Radius ist definiert als c/[H(t)]. Wäre der Hubble-Parameter konstant, wäre der Hubble-Radius ebenfalls eine Konstante.
    Die Lichtlaufzeit des Weltalters ist aber doch natürlich trotzdem für jedes Weltalter verschieden. D.h. für nur genau einen einzigen Zeitpunkt t würde der Hubble-Radius der Lichtlaufzeit des Weltalters entsprechen.

    In #179 habe ich diesen Zusammenhang schon geplottet.
    Für große Zeiten t wird das Verhältnis doch gerade deswegen immer schlechter, weil die Hubble-Entfernung gegen einen Grentwert läuft, also konstant bleibt, das Alter des Universums dagegen aber natürlich nicht.

  185. #185 Niels
    24. März 2015

    @Alderamin
    (Trennung wegen Spam-Filter)

    Zur Erinnerung noch mal die zeitliche Entwicklung der Hubble-Entfernung (in proper distance).

    denn die integrierte Expansion macht aufgrund der Dunklen Energie eine Schlangenlinie um die Gerade herum, die einem konstanten Hubble-Parameter entspräche, so dass die Gerade die Entwicklung für eine lange Zeit gut approximiert.

    https://www.wolframalpha.com/input/?i=Plot%5B16.2441+Tanh%5B0.0923534+t%5D%2Ft%2C+%7Bt%2C+0%2C+50%7D%5D
    Na ja, für den Zeitraum von 10 Milliarden Jahre nach dem Urknall bis 18 Milliarden Jahre approximiert es das Ganze mit einem Fehler nicht größer als 20%. Für größere t nimmt der Fehler sogar unbeschränkt immer weiter zu.
    Bei 14 Milliarden Jahren, also beim heutigen Alter, ist der Fehler dagegen ziemlich genau Null.

    Ist es Zufall, dass wir das Weltall nicht beim Alter von 6 bis 10 Milliarden Jahren beobachten, oder irgendwann zwischen 20 und 5000 Milliarden Jahre nach dem Urknall?
    Ich denke eher schon.

  186. #186 Niels
    25. März 2015

    @Alderamin
    Oh, deinen Beitrag #183 sehe ich erst jetzt. Musst du gepostet haben, während ich beim Tippen von #184 und #185 war?

    Ja, das macht deutlicher, was du meinst. Ich halt es zwar nach wie vor für einen “riesigen Zufall”, aber das läuft dann auf das Streiten über Bedeutungsnuancen hinaus.

  187. #187 Alderamin
    26. März 2015

    @Niels

    Ich hatte ohne die Formel zu betrachten (und mit falscher Erinnerung des Wiki-Artikels über den Hubble-Radius) im Hinterkopf, dass ein konstanter Hubble-Parameter zu einer linearen Vergrößerung des Hubble-Radius führt. Tatsächlich müsste der Hubble-Parameter sich dafür aber genau so entwickeln, dass eine Galaxie am heutigen Hubble-Radius zu jeder Zeit vorher die gleiche Fluchtgeschwindigkeit gehabt hat – die einfache Abschätzung des Weltalters aus dem Hubble-Radus, wie im Wiki-Artikel beschrieben, nimmt an, dass die Fluchtgeschwindigkeit konstant gewesen sei, aber dafür müsste der Hubble-Parameter eben früher höher gewesen sein, weil auf kürzere Entfernung normalerweise eine geringere Fluchtgeschwindigkeit besteht, und das auch noch in einer ganz bestimmten Weise.

    Falls es also keinen guten Grund dafür gibt, dass der Hubble-Paramter ohne andere Einflüsse (wie z.B. Dunkle Energie) sich genau so entwickelt hat, dass die Fluchtgeschwindigkeit in einer bestimmten Entfernung stets die gleiche war, dann ist es in der Tat ein großer Zufall, dass der Hubble-Radius heute fast genau so groß ist, wie die Lichtlaufzeitstrecke des Weltalters.

  188. […] Universum dehnt sich ja bekanntlich aus – und wie wir diese Woche gelernt haben, gab es dabei mit ziemlicher Sicherheit eine Zeit, in der diese Ausdehnung extrem schnell war. Aber […]

  189. […] mir – verkündete empirische Nachweis der so genannten Inflationstheorie (Florian hat dies sehr ausführlich beschrieben) im Zusammenhang mit dem Urknall vielleicht doch etwas anderes ist, nämlich ein […]